ANSWERS - Barron's AP Psychology, 7th Edition

Barron's AP Psychology, 7th Edition (2016)

ANSWERS

Diagnostic Test

ANSWERS EXPLAINED

1.(B) Easygoing, slow to anger, and relaxed are qualities of a Type B personality. If Rocco had an internal locus of control he would believe that he controls what happens to him. The Oedipus complex is the Freudian idea that boys desire their mothers and see their fathers as rivals for their mothers’ love. Temperament is one’s inborn style of relating to the world, and someone with an introverted temperament would be shy, unlike Rocco. Maslow and other humanistic theorists believe people have self-actualized when they have reached their full potentials.

2.(B) The experimental group is the one that gets the treatment involved in the independent variable; therefore, the group that listens to Mozart is the experimental group. It doesn’t matter whether the experimental group already listens to Mozart or has studied geometry so long as students are randomly assigned to the experimental and control groups.

3.(D) The space between two neurons is called the synapse. The nodes of Ranvier and myelin sheath both help increase the speed of neural transmission. Myelin is a fatty substance that insulates the axon, and the nodes of Ranvier are gaps between sections of myelin over which the impulse can jump. The axon carries the impulse between the soma and terminal branches. The medulla is the part of the brain that regulates involuntary behaviors such as breathing and heart rate.

4(B) Jobs in the early-twenty-first century are far more likely to involve sitting at a desk than they were 100 years ago, making it more difficult to burn calories at work. Significant changes in the gene pool are unlikely. Urban areas have not markedly increased in popularity, and people who live in cities tend to walk more than people who live in suburban areas. Working longer hours doesn’t make people gain weight, and there are an ever increasing number of ways to exercise.

5.(D) A learned taste aversion typically occurs when a novel taste (CS) is paired with an unpleasant reaction such as nausea (US). Negative reinforcement and shaping are terms generally associated with operant conditioning (learning by associating one’s behaviors with certain consequences). Insight learning typically occurs when one has a sudden realization about how to solve a problem.

6.(B) Divergent thinking is a term frequently associated with creativity. While convergent thinking involves the pursuit of one answer to a problem, often in a particular way, divergent thinking promotes more open-ended, innovative thought. Algorithms are formulaic approaches to problem solving that guarantee correct answers. Functional fixedness, the inability to think of a novel use for an object, is an obstacle to creativity. Having excellent recall ability is not necessarily related to being creative. Telegraphic speech, also known as the two-word stage of language, is common around age two.

7.(C) Arousal theory suggests that some people who have a high optimal level of arousal would pursue dangerous activities like skydiving. The other theories of motivation have difficulty explaining such actions.

8.(D) Authoritarian parents tend to make harsh rules and implement them without exception. Authoritative parents are more flexible with the creation and implementation of rules; this style of parenting has also been termed democratic. Indulgent and neglectful parents typically eschew rules, the former because they want to be kind and the latter because they don’t pay enough attention to their children’s needs.

9.(B) Calinda may suffer from dissociative identity disorder, formerly known as multiple personality disorder. This disorder, while questioned by some, manifests itself in breaks in consciousness and memory as the sufferer shifts from one personality to another. Conversion disorder is a type of somatoform disorder in which someone complains of a physical problem (e.g., deafness) for which no organic cause can be found. Schizophrenia, often confused with dissociative identity disorder, does not involve multiple personalities. Rather, someone with schizophrenia suffers from profoundly disordered thought. Post-traumatic stress disorder is an anxiety disorder that typically plagues people who have experienced tragic events like wars and natural disasters. Bipolar disorder is a type of affective disorder in which people alternate between periods of mania and depression.

10.(A) Psychoanalysts stress the pivotal role of childhood experiences and how they can manifest themselves later as anxiety and other types of problems. Biomedical psychologists are more likely to focus on the importance of biological factors such as hormones and neurotransmitters. Behaviorists believe that experience would have an important impact on anxiety, but they would not credit events from one’s youth as being particularly powerful. Cognitive psychologists emphasize the influence of the way people process information, while humanistic psychologists stress the effect of people’s needs and how they feel about themselves.

11.(A) Most clinicians are eclectic, meaning that they draw from a number of different perspectives.

12.(C) Since Curie, Madison, and Gandhi are all thought of as intelligent, an intelligence test on which they scored poorly would be criticized as lacking validity. A test that lacks validity does not test what it is supposed to test. Projective tests are typically used by psychodynamic psychologists to measure personality. It is possible for a test to be standardized, normed, and reliable and still not valid.

13.(D) The easiest and most common technique used to measure personality is a self-report inventory. Self-report inventories are typically questionnaires on which people answer close-ended questions. Some of the other methods are more difficult to employ (e.g., observing behavior and using brain scans). Projective tests, free association, and dream analysis are used mainly by psychoanalysts.

14.(E) Saluja is probably in the identity versus role confusion stage during which adolescents try on a variety of roles in an effort to define themselves. The other choices are all stages that Erikson proposed occur at other times during one’s life.

15.(D) Daniel is learning conservation of number, a skill that Piaget believed children learn in the concrete operational stage (ages 8–12).

16.(D) Seyle’s GAS (general adaptation syndrome) consists of three stages: alarm, resistance, and exhaustion. If a stressor wears a person out she or he reaches exhaustion and the parasympathetic nervous system returns the body to homeostasis.

17.(E) Procedural knowledge, your knowledge of how to perform skills such as tying your shoes, is thought to be stored in the cerebellum.

18.(A) It is easier for people and animals to learn new things when they are reinforced continuously. If something is reinforced every time, it is easier to form a link between the action and its consequences. Partial reinforcement schedules, however, are more resistant to extinction.

19.(C) Heroin is an opiate. Opiates are drugs that relieve pain. All the other drugs listed are stimulants.

20.(A) The stirrup is one of the ossicles, the three bones in the middle ear. The auditory nerve connects the cochlea to the brain. The cochlea is the structure in the inner ear in which the organ of Corti can be found. The pinna is the name for the fleshy part of the outer ear.

21.(C) Difference threshold, or just-noticeable difference, is the amount a stimulus needs to be changed in order for a person to detect a difference. Absolute threshold is the smallest amount of a stimulus necessary for one to detect its presence. If you were to add salt to a plain glass of water until someone could first taste it, you would be testing absolute threshold. Since the soup already had some salt, this question is about difference threshold.

22.(C) A CAT or CT scan uses X-rays taken from 180 different angles to create a computerized image that can depict the structure of the brain. MRIs can also show the structure of the brain, but they do so by using magnetic fields. An even more advanced technology is the fMRI, which uses magnetic fields to show both structure and function. EEGs show brain function by measuring electrical activity, and PET scans show brain function by measuring the metabolism of glucose.

23.(C) The purpose of Institutional Review Boards (IRBs) is to make sure researchers treat participants ethically. All research conducted at registered research institutions and/or intended for publication, must be approved by an IRB prior to any interaction with participants. None of the other people or groups listed in the choices would be permitted to stand in as a replacement.

24.(D) Basic research, as opposed to applied research, seeks to expand knowledge without a clear practical use. Choices A, B, C, and E all have clear, intended applications. Although learning about how children’s use of language changes could yield knowledge that would ultimately have a practical impact, such an application is not its immediate goal.

25.(B) Down syndrome is caused by an extra chromosome on the twenty-first pair. Klinefelter’s syndrome results when a boy has an extra X chromosome (XXY). Alzheimer’s and Tay-Sachs are due to genetic (not chromosomal) abnormalities. Fetal alcohol syndrome is caused by a mother drinking during her pregnancy.

26.(E) We know that when objects get closer to us they do not grow larger because, through experience, we have learned size constancy. Damage to the fovea would impair vision, and an inability to use binocular cues would limit depth perception. The Gestalt principles of perception (e.g., similarity, proximity) are thought to be inborn. Place theory is a theory about how we recognize pitch.

27.(C) Negative reinforcement is when a behavior (e.g., picking up a baby) is strengthened because it results in the removal of an aversive stimulus (e.g., crying). Insight learning, in this case, would involve parents suddenly realizing that holding their babies is a good thing. Positive reinforcement is when a behavior is strengthened by the addition of a pleasant stimulus; if babies applauded their parents when picked up, they would be positively reinforcing them. Latent learning is learning that occurs in the absence of reinforcement. Punishment is when a behavior becomes less likely due to the addition of an unpleasant stimulus; if we rephrased the question and asked how the parents were learning to avoid putting their babies down, the answer would be punishment.

28.(C) According to Noam Chomsky, children’s innate language acquisition device enables them to decode grammatical rules amazingly quickly. Sometimes, in the process, they apply the rules when it is incorrect to do so; that is, they overgeneralize. A child who understands the idea that the past tense in English is often denoted by the addition of -ed, therefore might add -ed to irregular verbs like go.

29.(A) Whorf’s linguistic relativity hypothesis holds that language influences (or, in its initial pronouncement, determines) thought. Therefore, referring to women as “girls” could affect the way people think about women. Bandura’s social learning theory explains how people learn by modeling the behavior of others. The nativist theory of language refers to Chomsky’s school of belief that human beings are wired in such a way that we learn language quickly and easily. Signal detection is a perceptual theory and arousal theory has to do with motivation.

30.(C) An approach-avoidance conflict is when one is attracted to and repelled by different features of the same thing. In this case, Mari is attracted to the idea of seeing Ben Folds but repelled by spending the evening with Jenna. In an approach-approach conflict, one must choose between two attractive alternatives. In an avoidance-avoidance conflict, one must choose between two unattractive alternatives. In a multiple approach-avoidance conflict, one must choose between several options each of which has an attractive and unattractive feature.

31.(E) Wundt’s goal was to identify the basic cognitive structures people used; hence, his theory is known as structuralism. William James’s theory of functionalism turned attention to how these structures function in our lives. Gestalt psychology focused on the importance of the whole. Trephination was an ancient practice of putting holes in people’s skulls in order to let evil spirits escape. Repression is a key part of Freudian theory.

32.(A) Trait theorists contend that personality is the expression of a person’s established characteristics, an explanation that makes it difficult to explain the effect that different situations can have on behavior. Behaviorists, conversely, stress the impact of the environment, and cognitive psychologists emphasize the role of one’s way of thinking; social-cognitive theorists believe that the interaction between environment and cognition gives rise to personality. Finally, psychodynamic theorists emphasize the role of unconscious processes and early childhood experiences.

33.(C) Heritability refers to the extent to which the variation of a factor in the population can be explained by genetic differences. Physical traits (e.g., hair color) tend to be more heritable than personality traits (e.g., conscientiousness) and there is virtually no evidence that attitudes (e.g., religious beliefs) are heritable at all.

34.(D) A test that measures potential is an aptitude test. A power test is comprised of items in increasing level of difficulty and is intended to ascertain the highest level at which one can perform, whereas a speed test contains many easy items and is meant to discern how fast one can solve the problems. Achievement tests measure what someone has learned. Individual tests, as opposed to group tests, are given to one person at a time.

35.(A) A standardization sample is a group of people who take a test to help establish norms and therefore standardize it. Introductory psychology students in college are thought to be similar to the high school students taking AP Psychology, so their performance is used to standardize the test. The other choices are made-up distractors.

36.(E) Somatoform disorders are marked by physical complaints in the absence of physical causes. A diagnosis of conversion disorder typically results when no organic (physical) cause can be found for a physical complaint such as paralysis of a limb, blindness, or deafness. Narcissistic personality disorder is, not surprisingly, a type of personality disorder; masochism is a paraphilia; generalized anxiety disorder is classified as an anxiety disorder; and psychogenic amnesia is a type of dissociative disorder.

37.(B) Dr. Hernandez’s belief that poverty can help trigger mental illness emphasizes the role of society and culture in psychological illness. Humanistic psychologists stress the importance of fulfilling one’s needs, biomedical psychologists focus on the impact of neurochemicals, cognitive psychologists put primacy on the influence of how one thinks, and behaviorists emphasize the effect of the environment.

38.(A) Door-in-the-face is a compliance technique in which one begins with a request that is likely to be perceived as too large and follows up with a smaller request that will surely be seen as more reasonable. Lowballing is when unattractive features of a decision are hidden until after someone agrees. Norms of reciprocity is the idea that people feel obliged to treat others as those others have treated them. Self-fulfilling prophecy is when one person’s expectations of someone else elicits behavioral confirmation in the second person. Foot-in-the-door is when one asks first for a trivial favor and then follows up with a larger request. That people acquiesce to the small favor makes it more likely that they will say yes to the more substantial second request.

39.(D) Cognitive dissonance theory posits that it is stressful to hold a thought (e.g., I hate my boss) that contradicts one’s actions (e.g., I am really nice to my boss). The stress motivates people to reduce the dissonance by bringing their beliefs into line with their actions. Therefore, since Elsa cannot change her behavior, she is likely to change her beliefs about her boss.

40.(C) ECT (electroconvulsive therapy) is most commonly used to treat depression. In some patients, severe depression that has not responded to drug therapy has been found to be relieved by ECT.

41.(E) Client-centered therapy is a type of humanistic therapy pioneered by Carl Rogers. Humanistic therapies emphasize the importance of the clients’ understanding of their problems, insight, and free will to change. Somatic therapies like psychopharmacology and psychosurgery and behaviorist techniques like flooding and token economies do not depend on or value the clients’ insights.

42.(D) Bystander intervention studies have shown that the fewer people around, the more likely it is that someone will help another person in need. One reason posited for this phenomenon is diffusion of responsibility, the idea that when others are present, any single person experiences a decreased feeling of responsibility to help. Interestingly, religious beliefs have not been found to predict helping behavior. People are more likely to help, however, when they are in good moods, an idea known as the “feel-good, do-good phenomenon.”

43.(A) One of the most common symptoms of depression is a disruption in eating habits, either by eating too much or eating too little. When depressed, people tend to lose interest in almost anything—work, hobbies, friends, and even family.

44.(E) Rogers asserted that we all need unconditional positive regard; unconditional positive regard comes from people whose love and support is not conditional on our behaving a particular way. Narcissism means self-love. Reciprocal determinism is Bandura’s theory that people’s traits, environments, and behaviors interact to shape their personalities. Thematic apperception is the name of a projective personality test. Self-actualization is Maslow’s idea that people strive to be the best they can be.

45.(A) Cross-sectional research seeks to identify the impact of aging by comparing different age groups at the same time. However, while it is possible that twenty-year-olds and fifty-year-olds differ due to the passage of time, it is also possible that they differ because twenty-year-olds were born in the 1980s and teens in the 1990s while fifty-year-olds matured in the 1960s; such differences are known as cohort effects. Choices B, C, and D are drawbacks to longitudinal research, and the effectiveness of cross-sectional research does not seem to be linked to the socioeconomic background of the participants.

46.(C) Achievement motivation is typically defined as a person’s drive to be successful in work or school. Achievement motivation is more closely linked to how good a job one seeks to do than to the type of job (e.g., doctor) that one has. People with high achievement seek to do their best, not merely to make money or gain other extrinsic rewards, but because they want to challenge themselves.

47.(B) Simon LeVay’s research has shown that a group of cells in the hypothalamus of heterosexual men is typically larger than the cluster in gay men, while part of the corpus callosum in gay men is typically larger than the same area in heterosexual men. Recent studies have debunked the myths that gay men and lesbians do not make good parents, were likely to have been abused as children, and that gay men have domineering mothers.

48.(E) Our tendency to approach problems in ways that have been successful for us in the past is known as mental set. Proactive interference is when something we learned first interferes with our ability to remember something we learned later. Functional fixedness is the tendency to overlook novel uses for items we are accustomed to using in a particular way. Belief bias is when people’s preexisting beliefs interfere with their logical reasoning. Framing is the finding that the way in which the same information is presented can impact the way we perceive it.

49.(C) The partial reinforcement effect is the idea that behaviors reinforced intermittently, as opposed to continuously, will be more resistant to extinction as it will be more difficult to unlearn the connection between behavior and reinforcement.

50.(B) People spend approximately 50 percent of their time asleep in stage 2. Approximately 25 percent is spent in REM, 20 percent in deep sleep (stages 3 and 4), and only about 5 percent in stage 1.

51.(B) Motion parallax is a depth cue. Nearby objects appear to move faster as we pass them than do objects that are far away. Texture gradient is also a depth cue; we can see more texture in objects that are close by than in those that are far away. Stroboscopic motion is what movies use to create the appearance of motion. The phi phenomenon explains why we perceive the light in a movie marquee or traffic sign as moving. Relative speed is a made-up distractor.

52.(A) Accommodation is the process by which the lens changes shape (flattens or thickens) to focus an image on the retina.

53.(E) The population is every case that could have been included in the study. Only the 362 new mothers who gave birth at Central Hospital could possibly have been selected. The 50 mothers with whom Farnaz speaks is the sample, not the population.

54.(D) A correlation shows a relationship but not necessarily a cause-and-effect relationship. Therefore, choices C and E are incorrect. A negative correlation is when the presence of one variable (e.g., hours spent meditating) predicts the absence of a second variable (e.g., stress); therefore, choice D is correct, and choice B is incorrect. Finally, choice A is wrong for several reasons including that it does not suggest an inverse relationship between the variables and that when research is done on a group of people, one cannot infer anything about particular individuals.

55.(B) During the early 1900s in the United States, behaviorism was the dominant psychological perspective. More recently, with technological advances that have made it possible to study the internal workings of the body and mind, the biological and cognitive perspectives have become increasingly prominent. Psychoanalytic psychology’s heyday was around the turn of the twentieth century, but it never achieved the prominence in the United States that it had in Europe. Gestalt psychology originated in Germany in the late nineteenth century.

56.(C) Standard deviation is measure of variability. The greater the standard deviation, the more varied or diverse the group. We would expect a lot of variability in the IQ scores of students in a large, public school system. Conversely, we would expect all the other groups to have less variability as they are narrower groups who are unlikely to differ from one another as much in terms of IQ.

57.(C) Insulin is a hormone secreted by the pancreas and used to turn glucose in the bloodstream into glycogen in the liver. Dopamine, endorphins, GABA, and acetylcholine are all neurotransmitters.

58.(B) Raising your hand to answer a question is a voluntary motor movement controlled by the somatic nervous system. The parasympathetic and sympathetic systems are the two parts of the autonomic nervous system; they control involuntary functions such as heart rate and digestion. The central nervous system includes the brain and spinal cord. Although they would have played a role in telling Gonzo to raise his hand, taking that action was ultimately enabled by the motor neurons in his somatic system.

59.(B) The hypothalamus controls motivated behaviors such as eating and having sex. The amygdala is important in fear and other emotions. The pituitary gland is the master gland of the endocrine system. The medulla is the part of the brain that regulates involuntary processes like breathing. The hippocampus is important in memory.

60.(C) Bottom-up processing is when an object is perceived only by examining the object itself. Signal detection theory and perceptual set are theories about how one’s expectations and past experiences can impact perception. Since Olivia’s experience is limited to the concrete jungle of New York City where cows are unusual and she saw the cow in a place one would not have expected to see a cow, her perception of the cow would have relied on bottom-up processing. Difference threshold is the amount a stimulus needs to change in order for someone to perceive that it has changed. Brightness constancy is our knowledge that the relative brightness of objects stays the same even as the level of overhead illumination changes.

61.(C) The opponent process theory of color vision explains that some of the cells that help us to see color are organized in opponent pairs: red and green, blue and yellow, black and white. If we fatigue one half of the pair and then look at a white surface that reflects all wavelengths of light, we will see an image in the opponent colors (a negative color afterimage). Hence, the red of the original parrot is seen as green and the yellow appears blue.

62.(C) Latent learning is learning that occurs in the absence of a reward even though it may not be evidenced until a reward is present. Tolman had three groups of rats: one that was rewarded every time they finished a maze, one that was never rewarded, and one that was only rewarded on the second half of the trials. Tolman found that this third group performed like the unrewarded group until the reward was introduced and then showed a tremendous improvement as soon as they could earn a reward. He concluded that the rats had learned during the first half of the trials but simply not had any reason to hurry to finish the maze since there was no reward; thus, their learning had been latent or hidden.

63.(E) Rescorla’s contingency theory of classical conditioning modified the original contiguity theory by saying that learning didn’t simply depend on the number of times a CS and US were paired but rather the degree to which the CS reliably predicted the US. The US can be said to be contingent upon the CS when the CS reliably predicts the US.

64.(E) Memories are stored all over the brain, which casts doubt on recent movies in which people can get unpleasant memories erased. People do not have a good sense of whether or not their memories are accurate. Young children’s brains are not developed enough to form clear, explicit memories. Older people are worse than young people at some types of memory tasks such as recall tasks but not all; remember, exercise caution before choosing extreme answer choices that contain words like all. Contrary to what many people believe, memories are not like video images we can replay at will; rather, we may reconstruct and alter them each time we recall them to mind.

65.(C) Maslow’s hierarchy of needs begins with physiological needs, moves to safety needs, belongingness needs, esteem needs, and, finally, self-actualization. Since Kelsey can support herself and lives in a relatively safe place, the first two levels of need are probably fulfilled. Her many friends probably fulfill her need to belong. Kelsey is struggling to make a name for herself as an actress, which reflects a need to achieve something and be respected; these are esteem needs.

66.(D) The Moro reflex is also known as the startle reflex. The Babinski reflex is babies’ tendency to spread their toes when the bottom of the foot is scratched; the plantar reflex is the opposite tendency in adults. The rooting reflex is when, in response to feeling a touch to its cheek, a baby will turn its head to find a nipple. Grasping is when the infant will curl its finger around an object placed in its palm.

67.(E) Newborns love to look at human faces. Real faces are preferable to most anything.

68.(B) Reaction formation is a defense mechanism in which one expresses the opposite of what one feels; Roscoe feels hate but professes love. Displacement is when one takes out one’s feelings on a less threatening target than the cause. Projection is when one attributes one’s own undesirable traits to others. Sublimation is when one channels one’s sexual energy into more acceptable pursuits. Intellectualization is when one deals with a problem in a distant, detached manner.

69.(E) Ninety-five percent of the scores in a normal distribution fall between the z scores of –2 and +2. Z scores measure the distance from the mean in units of standard deviation. The percent of scores falling under each area of the normal curve is predetermined; there’s no way around memorizing it: 68 percent within one standard deviation, 95 percent within two standard deviations, and 99 percent within three standard deviations.

70.(C) The age of onset for schizophrenia is usually in the early 20s.

71.(D) Bipolar disorder has been linked to an excess of acetylcholine receptors. The manic phase responds well to lithium, and the depressed phase responds well to antidepressant medications. In addition, much research suggests that bipolar disorder runs in families. Though we may be predisposed to develop certain phobias more easily than others, phobias are well explained by and treated from other perspectives, particularly the behaviorist perspective. The existence of dissociative identity disorder is hotly debated; those who believe in its existence generally posit a traumatic event in childhood as a cause. Conversion disorders have become increasingly uncommon in the United States; such a change argues against a genetic cause as the gene pool remains relatively stable. Further, a conversion disorder involves a physical complaint in the absence of a physical symptom and is therefore unlikely to have a biological cause.

72.(D) The false consensus effect is the tendency of people to overestimate the number of people who share their views. Deindividuation is a loss of self-restraint under conditions of anonymity and arousal. The just-world bias is people’s belief that good things happen to good people, and bad things happen to bad people. Norms of reciprocity is the idea that we feel obligated to treat others as they have treated us. Outgroup bias refers to the negative attitudes people often have toward people who do not belong to the same groups that they do.

73.(B) Dr. Lupin’s approach is a cognitive-behavioral one; she challenges her clients’ beliefs (cognitive) and gives them homework assignments to practice new behaviors (behavioral). Rational emotive behavior therapy (REBT), developed by Albert Ellis, does just that. Modeling is the closest alternative, but it involves the observation and imitation of behaviors. Existential and Gestalt therapies are humanistic approaches. Psychoanalysis involves probing patients’ unconscious in order to discover the repressed roots of their complaints.

74.(C) Milgram showed that the majority of participants were willing to deliver an electric shock of sufficient voltage to kill someone; thus, most were willing to do something immoral and illegal. Members of minority groups were not compared to Caucasians. No hypnosis was used. Women and men behaved similarly.

75.(D) If Artie acts competitively in situations where others do not (low consensus), people are likely to attribute his behavior to something in himself (a person attribution) rather than something in the situation. If Artie always acts this way (high consistency), people are likely to make a stable (as opposed to unstable) attribution.

76.(C) Group therapy tends to be less expensive than individual therapy as the cost is shared by the members of the group. Some people may make friends with others in their group, but it is highly unlikely that they develop lasting friendships with ALL the other people in their group and making such friendships is not the goal of seeking therapy. Another asset of group therapy, albeit not a choice in this question, is that people come to understand that many people have the same kind of problems as they do. Few clear differences exist between group and individual therapy in terms of success rate, length of therapists’ training, and time spent in therapy.

77.(A) Behaviorist therapies tend to be relatively brief and focus on replacing behaviors that make clients unhappy with ones that will make them happier. Psychodynamic therapies tend to be long, in part, because they involve a search for the underlying cause of a patient’s issues, a search that often involves a discussion of someone’s entire life. Since people from the sociocultural perspective see society as the cause of mental illness, short-term therapy that focuses on the individual is difficult. Humanistic therapies tend not to be as focused as behaviorist ones; they may focus on more abstract issues such as how to find meaning in one’s life rather than on how to alter a particular behavior. Somatic therapies are not psychotherapies; most commonly they involve changing one’s biochemistry via drugs.

78.(D) Hostile aggression is aggression for its own sake rather than aggression that is instrumental in attaining a goal. In all the other choices, the goal of the aggression is to gain something (e.g., supper, money). In choice D, the aggression is intended to accomplish nothing other than serve as an outlet for Tutti’s anger.

79.(A) Howard Gardner has posited the existence of eight different types of intelligence (naturalist, bodily-kinesthetic, musical, interpersonal, intrapersonal, spatial, linguistic, and logical-mathematical). Practical and experiential are types of intelligence discussed by Sternberg. Fluid intelligence (as opposed to crystallized intelligence) tends to decline as one ages. General intelligence (g) is Spearman’s concept of a single factor that underlies all types of intelligence.

80.(C) There are two kinds of trait theorists: idiographic theorists who believe that we need different sets of traits to describe different people and nomothetic theorists who believe all people can be described with one set of fundamental traits. None of the other choices refer to trait theorists.

81.(C) Drinking while pregnant may lead the baby to be born with fetal alcohol syndrome (FAS). FAS children tend to suffer from mental retardation. They are not born addicted to alcohol and are unlikely to become drug dealers. The main cause of low birth weight is mothers who smoke. FAS children are not particularly likely to develop heart disease.

82.(C) Harry Harlow studied baby monkeys and is credited with demonstrating the importance of contact comfort. Harlow’s work suggested that infants crave the physical comfort of a body. Gilligan is known for her work challenging Kohlberg’s theory of development as gender biased. Piaget developed a stage theory of cognitive development. Freud’s best-known contributions are his stage theory of psychosexual development and treatment technique of psychoanalysis.

83.(A) Isabella’s memory of the first time she went skydiving is a declarative or explicit memory because it is a conscious memory she can actively recall. It is also an example of an episodic memory, but that term is not a choice. Semantic memory is our memory for facts. Implicit memory, as opposed to explicit memory, contains memories we don’t even realize we have. Eidetic memory is another name for photographic memory. Procedural memory is memory for how to do things such as take a foul shot or play the piano.

84.(A) The James-Lange theory of emotion asserted that events cause distinct physiological responses in our bodies that were then recognized as particular emotions. For instance, hitting a home run alters your body in a specific way that tells you that you feel excited and happy. This theory is often confused with Schachter’s two-factor theory described in choice E. Schachter’s theory holds that the physiological response to most events is similar, a general kind of arousal. The particular emotion is discerned by a cognitive process of appraisal that follows. The Cannon-Bard theory posits that the thalamus plays a central role in the process of identifying emotions. Choice C describes the opponent process theory of emotion. Choice D refers to Paul Ekman’s work that showed that throughout the world most people recognize emotions similarly; for example, a smile is seen as a sign of happiness.

85.(D) Observational learning is when one member of a species observes a behavior in another member of that species and then copies it. The same species aspect of the definition means that a girl howling like a wolf and a parrot imitating its owner saying “mama,” are not examples of observational learning. The other two choices involve no observation or imitation.

86.(B) Discrimination in operant conditioning is learning that a behavior will only result in reinforcement under certain conditions. Since earthworms are easier to find after it rains, Melissa has learned to discriminate between the good and bad times to dig for earthworms.

87.(D) In a normal distribution, the mean, median, and mode are equal. Ninety-five percent of the scores fall within two standard deviations of the mean. The normal curve is asymptotic, which means it never hits the axis so we cannot say that everyone falls within any number of standard deviations from the mean. The standard deviation can be less than or more than 1; it depends upon what is being measured.

88.(A) When a neuron depolarizes, sodium ions flow into the cell. Sodium ions carry a positive charge, and their influx helps change the charge of the cell from negative to positive. The negative charge of the cell at rest is due, largely, to the presence of many chloride ions in the cell; this number does not increase during depolarization. Potassium ions also carry a positive charge and flow into the cell when it is depolarized. Neither magnesium nor strontium plays a role in this process.

89.(D) The prefrontal cortex plays an important role in decision making. Decision making is a highly complex cognitive skill, which should serve as an indication that the answer will probably involve the cortex.

90.(D) Experimenter bias refers to the idea that researchers’ beliefs in their own hypotheses may cause them inadvertently to influence the results of the research so as to confirm those hypotheses. Confirmation bias refers to a similar tendency in all people to pay more attention to information that supports their preexisting beliefs than to information that refutes them. The availability heuristic is the tendency to draw conclusions about the frequency of something based on how easy it is to recall it to memory. Functional fixedness is the tendency not to recognize that a familiar object can be used in a novel way. The representative heuristic is the tendency to reason by similarity and, in the process, to underweight base rate probability. For instance, people might believe that a tall, very thin, attractive woman would be more likely to be a supermodel than a librarian. Overconfidence is people’s tendency to be excessively confident in their decisions.

91.(B) Iconic memory is the name given to our visual, sensory memory. It has a large capacity but lasts only for a fraction of a second. Short-term memory holds only about seven items and lasts for about 30 seconds. Long-term memory is thought to be essentially limitless in both capacity and duration. Echoic memory is the auditory sensory memory; it lasts three to four seconds. When we are actively processing information, it is in our working memories.

92.(A) Objects appear the color of the wavelengths of light that they reflect.

93.(C) Your knowledge that George Washington was the first president of the United States is usually found in your preconscious. Now that I have caused you to think about Washington, the information is in your conscious. The nonconscious controls your body processes such as heart rate and digestion. Information in your subconscious affects how you process information and includes implicit memories. The unconscious is a term used mostly by psychodynamic theorists to refer to troubling thoughts that we have actively pushed out of our conscious minds.

94.(E) Freud suggested that, while going through the psychosexual stages, people could get fixated. People who have phallic fixations typically have issues of either over or underconfidence about their sexual prowess. A strong libido would cause Karl to seek dating, or at least sex, frequently. The ego is guided by the reality principle. The preconscious refers to information about which one is not thinking but that could be called to the conscious mind easily. Someone with an anal expulsive personality is usually extremely messy and disorganized.

95.(E) Scores on the WISC are normally distributed. The WISC has a mean of 100 and a standard deviation of 15. Banu, therefore, scored two standard deviations above the mean: (130 – 100)/15 = 2. Z scores are a measure of the distance from the mean in units of standard deviation, so Banu has a z score of +2, making possible answers D and E. Percentile is a measure of the percent of test takers who scored at or below a particular score. We know that 50 percent of the test takers scored at or below the mean. We know that an additional 34 percent of scores fall between the mean and one standard deviation above the mean, and we know that another 13.5 percent of scores fall between one and two standard deviations above the mean. Adding these numbers together tells us that Banu scored at the 97.5th percentile, making the answer E.

96.(E) Positive symptoms of schizophrenia are ones that are related to excesses rather than deficits. Having hallucinations, perceiving sensory stimulation when none exists, is a positive symptom. Flat affect and catatonia are negative symptoms since they are deficits in emotion and movement, respectively. Greater sensitivity to others and reduced depression are positive things, but they are not symptoms of schizophrenia.

97.(A) Broca’s area is typically in the left frontal lobe. Broca’s area controls our ability to use our mouths to form the words we wish to say. In most people, language functions are controlled mostly by the left hemisphere. The frontal lobe has many important functions relating to movement; it also houses the motor cortex.

98.(E) GAD, generalized anxiety disorder, results in the client experiencing a constant, low-level feeling of tension. Such feelings can be treated with mild sedatives such as Valium. Tricyclic antidepressants are most often used to treat depression. Thorazine and Haldol are typically used to treat schizophrenia. Lithium has been shown to be helpful in treating the manic phase of bipolar disorder.

99.(E) Group polarization is the phenomenon that, given time together to discuss something, groups of like-minded individuals will often come to hold more extreme ideas than those with which they entered the group. One possible reason for this phenomenon is that in the group, the responsibility for the extreme decision seems to be divided among the group’s members. Out-group bias is the prejudice people feel against members of other groups. Self-fulfilling prophecy is the idea that one person’s expectations about another person can influence the second person’s behavior. Self-serving bias is the tendency to take greater responsibility for successful outcomes than unsuccessful ones. The fundamental attribution error is the tendency to underestimate the influence of situational factors on other people’s behavior.

100.(A) In a positively skewed distribution, there are more low scores than high scores. Typically, one or several unusually high scores, or outliers, skews the distribution. As a result, the mean is pulled up, toward the extreme scores, and is therefore higher than the median. Outliers generally have no effect on the mode, the most frequently occurring score, so it is difficult to make a general statement about the relationship of skew to the mode.

History and Approaches

ANSWERS TO PRACTICE QUESTIONS

1.(C) Scientific research is empirical by nature, and Wundt based the theory of structuralism on results of experimentation. Wundt’s reputation and the academic setting are not relevant to the scientific nature of his theory. Wundt did not study under Pavlov nor was he a clinical psychologist.

2.(E) The unconscious mind contains memories of events or feelings of which we are not consciously aware. It was not the first comprehensive theory. Freud did not use cadaver dissection to formulate the theory. While the sex drive figures prominently in the theory, the theory is not focused entirely on sex. While some psychologists would dispute much of Freud’s theory, many therapists still find the idea of the unconscious mind relevant.

3.(D) Behaviorists look at what behaviors we are rewarded for, and cognitive psychologists explain our behavior through the way we interpret events. The rest of the answers are explanations of other psychological perspectives incorrectly applied to behaviorism or cognitive theory.

4.(B) The cognitive perspective emphasizes the role of interpretation of others’ actions and best fits the given scenario. The other perspectives would emphasize other types of explanations.

5.(A) Wundt used the technique of introspection to research his theory of structuralism. He did not use naturalistic observation and inferential and scientific are general terms that might be applied to many different types of research.

6.(A) William James wrote the first psychology textbook, The Principles of Psychology, in 1890.

7.(E) Wertheimer is the only psychologist in this list that was included as part of the Gestalt group in the text.

8.(C) The unconscious mind is an integral concept in Freudian theory. Memories and impulses are repressed into the unconscious mind and this drives our later behaviors. The other concepts mentioned do not relate to psychoanalytic theory.

9.(D) Freud did not use the scientific method and many of his conclusions cannot be tested. The theory was based on individual male and female cases in therapeutic settings, and is not tied closely to behaviorism.

10.(D) Watson referred to the conditioning experiments Ivan Pavlov did with dogs. B. F. Skinner came after Watson chronologically, and the other psychologists mentioned are not part of the behaviorist paradigm.

11.(B) Skinner added the idea of reinforcing events, such as rewards and punishments, to the basic idea of behaviorism. Unconscious thinking and defense mechanisms are concepts from the psychoanalytic perspective. Conditioning is a general term used before Skinner did his research. Introspection was a technique used by Wundt.

12.(C) Maslow is the only psychologist in this list included in the section on humanism in the text. Skinner, Pavlov, and Watson are behaviorists.

13.(C) The idea that dreams contain symbols is central to the psychoanalytic perspective, and not any of the other perspectives listed.

14.(A) Behaviorists explain human thought and behavior as a result of conditioning in our pasts, either classical conditioning or operant conditioning. This conditioning restricts (or eliminates) the idea of personal choice, and behaviorists do not usually refer to biological causes and evolutionary theory (like natural selection). The unconscious mind is a concept from psychoanalytic theory.

15.(C) The eclectic perspective claims that no one perspective can best explain all human behaviors and a therapist who works with many different perspectives to help individual patients is in some sense eclectic, not any of the other specific perspectives listed.

Methods

ANSWERS TO PRACTICE QUESTIONS

1.(B) Psychologists generally prefer the experimental method to other research methods because experiments can show cause-effect relationships. The hallmarks of an experiment are the ability to manipulate the independent variable, randomly assign subjects to conditions, and eliminate (control for) differences between the conditions. When these steps are taken, disparities between the experimental and control groups can be attributed to the independent variable, the only thing that differed between the groups. No other research method allows for the control necessary to make such an attribution. None of the other statements are true.

2.(D) Random assignment should eliminate subject-relevant confounding variables (e.g., conscientiousness, IQ, hair color). Since it would be impossible to match participants on every possible dimension, many psychologists use random assignment. By taking advantage of the laws of probability, random assignment makes it likely that participants in the different conditions of an experiment will be equivalent. Random assignment does not relate to sampling or validity and has no impact on the need for statistics.

3.(A) Charlotte and Tamar’s way of measuring friendliness is not reliable. Reliability refers to the consistency of a measure. Since they disagree so often, their measure is not consistent. In all likelihood, they need to operationalize their dependent variable more clearly. Reliability is sometimes confused with validity. Validity refers to the accuracy of a measure, which in this case is whether they are actually measuring friendliness.

4.(E) It would be extremely difficult to test whether parental divorce causes children to become more independent because it is essentially impossible to manipulate the independent variable, divorce. If we tried to recruit parents to be in our experiment and told them that half would be assigned to divorce and the other half to stay married, it is unlikely that any would consent to participate. If you wanted to compare children whose parents had divorced with children from intact families, you could conduct a quasi-experiment. In a quasi-experiment, the researcher is unable to manipulate the independent variable but tries to control as many other factors as possible. The other four examples are relatively easy to study experimentally as the independent variables (color, exercise, exposure to violent television, and studying) are not difficult to manipulate.

5.(D) Professor Ma would need to be particularly careful to ensure the participants’ confidentiality in this study since it deals with a controversial and possibly embarrassing subject. There is no indication that Professor Ma coerced the participants or deceived them in any way. Confounding variables are a concern for the validity of the study but not an ethical consideration. The study does have a clear scientific purpose.

6.(B) Milgram’s experiments involved considerable risk of long-lasting stress and anxiety for his subjects. The scientific purpose of Milgram’s study is not disputed, and he debriefed participants and provided for anonymity of individual results.

7.(B) Within the limits imposed by the guidelines, researchers can physically harm animals if the harm is justified by the nature of the experiments. Deception is obviously not an issue applicable to animal research. Researchers must keep suffering to a minimum, so “any amount” is not an appropriate response (choice D). Animal subjects must be chosen carefully (from accredited commercial sources) and their environment must follow strict guidelines, so choices C and E are also incorrect.

8.(E) Lily’s z score is +3. Z scores measure the distance of a score from the mean in units of standard deviation. Since the mean is 100 and the standard deviation is 15, Lily’s score is 3 standard deviations above the mean.

9.(B) The median of the distribution is 5. The problem is easier if you put the scores in order: 2, 3, 4, 6, 7, 9. Since the distribution has an even number of scores, there is no middle score and you must average the two middle scores, 4 and 6.

10.(C) Emma’s perfect score is an outlier and will therefore skew the distribution. Since it is a high score in a distribution of low scores, the distribution will be positively skewed.

11.(D) José needs to compare the performances of the two groups using inferential statistics to determine whether or not the experimental group’s performance was significantly better. Scatter plots are used to graph correlations. José would certainly be interested in descriptive statistics as well, but he would not know whether or not his hypothesis had been supported until he used inferential statistics. Histograms are bar graphs, and means-end analysis is a problem-solving technique.

12.(B) Of the three methods presented, only having a computer generate a random list of names is an example of random sampling. The first example illustrates random assignment and not random sampling. Sampling is the process of choosing a group of participants from a population. Once sampling has been completed, one might assign the participants to conditions as described in I. Finally, approaching 50 students during a lunch period does not constitute random sampling even if the person who picks the people tries to do so randomly. Remember that the word random has a very specific meaning in the context of research. Random sampling means that all members of the population had an equal chance of being selected, and people are unable to be so scrupulously unbiased.

13.(C) A confounding variable is anything that differs between the control and experimental group besides the independent variable. How fast and frightened the mice are at the onset of the experiment are potential participant-relevant confounding variables. When and where the experiment takes place are possible situation-relevant confounding variables. However, the population from which Vincenzo selected his mice is not a confounding variable; they all came from the same population. True, the population can be flawed. For instance, it can be very homogeneous and thus fail to reflect how other mice would perform under similar conditions. However, such a flaw is not a confounding variable.

14.(D) Olivia is using naturalistic observation. As a student herself, she can observe the finger-painting table unobtrusively. She does not interact with the finger painters; she merely observes. Because Olivia does not manipulate an independent variable nor attempt to control any aspect of her study, she is not using any kind of experiment. She did not ask the participants questions as she would have if she were conducting a survey. She did not focus on a single participant or a small group of participants as she would have if she had been interested in putting together a case study. Finally, Olivia has not conducted ethnographic research. Ethnography is a type of research in which the researcher immerses himself or herself in another culture and then describes it. Ethnography is a method most commonly employed by anthropologists.

15.(B) Talia has established a relationship, or correlation, between the two variables she is studying. However, since she has not conducted an experiment, Talia does not know whether a cause-and-effect relationship occurs between studying and earning high grades on the exam. Therefore, Talia does not know if studying improves exam grades. Although Talia has found a correlation between studying and exam grades, whether or not that correlation is significant can be determined only through the use of inferential statistics. Even if the correlation were significant, it would not guarantee that if someone did not study, he or she would do poorly on the test. Finally, Talia’s correlation does not tell us that better students study more. In fact, it tells us nothing about better students, not even what is meant by that term.

Biological Bases of Behavior

ANSWERS TO PRACTICE QUESTIONS

1.(D) The visual cortex is located in the occipital lobe. The other locations are incorrect for the visual cortex. The sensory cortex interprets touch stimuli, and the cerebral cortex is the term for the entire wrinkled surface of the brain, so those items are incorrect.

2.(B) The motor cortex (which is located in the frontal lobe) in the right hemisphere controls the left side of the body. No such thing as the sensorimotor cortex exists, and the other locations for the motor cortex are incorrect.

3.(B) The auditory cortex is located in the temporal lobe and is connected to the inner ear by the auditory nerve. Other locations given for the auditory cortex are incorrect. The sensory cortex, hypothalamus, and cerebellum are not involved in hearing.

4.(B) The old or reptilian brain exists in all mammals and is thought to have developed first. As humans evolved into primates, the cerebral cortex developed and grew larger, allowing us to solve more complex problems. All brain structures are present in children from birth. All parts of the brain might deal with new or old information. Dementia is not more likely to affect the old or new brain.

5.(D) Excitatory neurotransmitters increase the likelihood that the next neuron will fire. Inhibitory neurotransmitters actually decrease the chance the next neuron will fire when received by the cells’ dendrites. Synaptic peptides and adrenaline-type exciters are not relevant to neuroanatomy (or any other anatomy—they are nonsense terms!). Potassium and sodium are integral in the process of depolarization but are not secreted from terminal buttons into the synaptic gap.

6.(C) Acetylcholine is the neurotransmitter involved in muscle control. Serotonin is also a neurotransmitter, but it would not be responsible for losing control over your muscles. Insulin is involved in hunger control. Thorazine is an antipsychotic drug prescribed by psychiatrists. Adrenaline is a hormone released by the adrenal glands in response to stressful situations.

7.(C) The hindbrain, midbrain, and forebrain are three of the traditional categories of brain structures. The new brain is synonymous with the cerebral cortex. The brain stem, limbic system, and cerebral cortex are divisions of the brain but not overall categories that include the entire brain. Neurons and synapses are parts of neuroanatomy, not major divisions of the brain.

8.(B) Spinal reflexes, such as the reflex that causes your leg to move when a doctor strikes your leg just below your kneecap, are controlled by the spine, not the brain. Stress is not relevant to the process, nor are afferent and efferent nerves. All spinal reflexes involve the peripheral nervous system. Humans do have some spinal reflexes; they are not limited to animals.

9.(A) Serotonin is the only neurotransmitter on the list that is identified as being involved in mood disorders. Adrenaline is a hormone released by the adrenal glands in response to stressful situations. Acetylcholine is a neurotransmitter that controls muscle movements. Endorphins are painkillers in the brain that might temporarily elevate mood but would not be responsible for long-term mood disorders. Morphine is a drug that interacts with our endorphins to alleviate pain.

10.(A) Neural firing involves an electric charge within the cell and chemical transmission between cells (across the synapse). It is electric within the cell and chemical between the cells. The electric charge does not jump the gap between neurons. The question refers to an individual neuron firing, not a group of neurons. Neurotransmitters are not confined to the hindbrain or the forebrain.

11.(C) The PET and EEG scans both give information about brain function (the PET measures brain activity, and the EEG measures brain waves). The MRI and CAT scans give information about brain structure, not function. An EKG is a medical test for heart function. Lesioning involves destroying brain tissue and would not be used in this type of research.

12.(B) Since the left hemisphere typically controls speech, split-brain patients are usually unable to talk about information exposed to only the right hemisphere. Their muscle coordination is usually normal (possible after a short adjustment period). Their ability to solve abstract problems is not affected. Visual information from the left part of each eye is transmitted to the right hemisphere, not the entire left eye. Both hemispheres receive auditory information from the left ear.

13.(E) Plasticity refers to the brain’s ability to rewire itself to recover functions lost through some type of brain damage. This process occurs most quickly in children but can happen to a limited extent in adults. The brain does not quickly regrow neurons, it reconnects existing neurons in new ways. Plasticity has little to do with the texture and appearance of the cerebral cortex. The adaptability referred to in choice D is related to plasticity, but the correct answer E is a much more specific explanation.

14.(C) A tumor on the hypothalamus would explain the first three symptoms since the hypothalamus controls (at least in part) body temperature, libido, and hunger. The cerebellum coordinates some types of movements, including throwing objects and our sense of balance. The motor cortex controls voluntary muscle movements, but the specific movements described in the question are controlled by the cerebellum. The medulla controls our life-support functions, like heart rate and respiration. The thalamus directs signals coming in from the spinal cord to different parts of the brain.

15.(A) As mentioned previously in the analysis of split-brain patients, the left hemisphere typically controls speech. The left hand is controlled by the motor cortex in the right hemisphere. Some evidence indicates that the right hemisphere is more active in spatial reasoning. Both hemispheres are involved in hypothesis testing and abstract reasoning.

Sensation and Perception

ANSWERS TO PRACTICE QUESTIONS

1.(B) A nerve connects the olfactory bulb directly to the amygdala and hippocampus. This connection may explain why smell may be a powerful trigger for emotions and memories. This connection has nothing to do with learning, long-term memory, or deep processing. Smells are eventually communicated to the cortex, but that does not explain the special connection to memory.

2.(C) Hair cells inside the cochlea change the mechanical vibrations received at the oval window into neural signals that are transmitted to the brain. The cochlea is part of the ear, not the eye, so choices A and D are incorrect. The hammer, anvil, and stirrup transfer vibrations to the cochlea, not the other way around. The semicircular canals send messages to the brain about the orientation of the head and body.

3.(E) According to shape constancy, we know shapes remain constant even when our viewing angle changes. This experiment would not be investigating feature detectors, because the equipment required to measure the firing of feature detectors is not described. Placement of rods and cones in the retina would not affect perception of the top of the box. Binocular depth cues are probably not the target of the research because the researchers are not asking questions about depth.

4.(A) The spot where the optic nerve connects to the retina lacks rods and cones and is thus called the blind spot. Choices B and C are distracter items and are not true. Floating debris and retinal damage could cause blind spots. However, these do not occur in everyone, and the question implies the blind spot present in everyone’s eyes.

5.(B) We sense tastes and smells by absorbing chemicals. Energy senses are hearing, sight, and touch. Flavor senses and memory senses are not valid terms. Choice D is incorrect because all nerve impulses are sent by an electrochemical process.

6.(C) Amplitude is a measure of the height of the wave, creating the volume of the sound. Frequency is the measure of how quickly the waves pass a point, causing the pitch of the sound. The other choices are incorrect distractions.

7.(E) Weber’s law calculates the difference threshold or the just-noticeable difference. It has nothing to do with sight, subliminal messages, or amplitude.

8.(B) Gate-control theory explains why some pain messages are perceived while others are not. This theory is specific to the sense of touch, so choices A, C, and D are incorrect. Choice E is incorrect because gate-control theory has to do with the perception of pain, not how we interpret sensations in general.

9.(B) All the other choices are monocular cues for depth, so they could be used by a person sighted in only one eye. Convergence is a binocular cue and would not work without two functioning eyes. When an object is close to our face and our eyes have to point toward each other slightly, our brain senses this convergence and uses it to help gauge distance.

10.(D) Sensation is the activation of our senses by stimuli, and perception is how we organize and interpret sensations. Choice A is incorrect because some sensation processes are more than mechanical. Choice B is too vague—advanced in what sense? Some researchers think part of perception may happen in the senses themselves, so choice C is incorrect. Choice E is false; perception involves learning and expectations.

11.(A) Visual receptors, rods and cones, are embedded in the retina, which is the back part of the eye. The rest of the items are incorrect because they describe functions the retina does not perform.

12.(C) The opponent-process theory explains these two phenomena, which the trichromatic theory cannot do. Visible hue is not a color vision theory. Dichromatic is a type of color blindness, not a theory of color vision. Binocular disparity is a depth cue.

13.(C) Your mind filled in the information from the picture by drawing on your experience. This is top-down processing. The example does not reflect bottom-up processing because information is being filled in, instead of an image being built from the elements present. Signal detection theory has to do with what sensations we pay attention to, not filling in missing elements in a picture. Opponent-process theory explains color vision. Gestalt theory might relate to this example because you are trying to perceive the picture as a whole, but there is no such term as gestalt replacement theory.

14.(D) Our vestibular sense helps with our sense of balance and orientation in space. Our vestibular sense has little to do with our sense of sight or hearing. Repeating digits would not be affected by the vestibular sense. Our kinesthetic sense gives us information about the position of our limbs.

15.(C) Most perceptual principles apply in all cultures. However, some perceptual sets are learned and will vary, so choices A and B are incorrect. Sensory apparatuses do not vary among cultures, and perception is not genetically based as implied in choice E.

States of Consciousness

ANSWERS TO PRACTICE QUESTIONS

1.(B) Agonists fit into receptor sites for specific neurotransmitters and produce similar results. Choice C is a definition of antagonists. The other choices are incorrect distractions.

2.(C) Babies spend more time in REM. As we get older, the time spent in REM gradually decreases. The other choices are incorrect statements about the typical sleep patterns of infants.

3.(D) Psychologists define consciousness as our level of awareness of ourselves and our environment. A dimmer switch is the only analogy that implies a continuum from very dim to very bright with variations in between. Consciousness is not like an on/off switch as implied in the other choices.

4.(C) Most often, we cycle through the sleep stages around 5 to 6 times per night. The duration of a sleep cycle is approximately 90 minutes long.

5.(A) People who have richer fantasy lives are more easily hypnotizable. This finding supports role theory, the idea that people may be acting out a social role under hypnosis. Choice B is irrelevant and incorrect according to research. Hilgard’s hidden observer is evidence for the dissociation theory of hypnosis. Choices D and E are true but are not evidence for role theory.

6.(C) Activation-synthesis theory states that dreams are a meaningless by-product of brain processes during REM sleep. The other choices do not relate to this theory.

7.(D) The hidden observer indicates that hypnosis might involve a dissociation of consciousness into different levels. Hilgard’s experiments are evidence against the other theories of hypnosis, role and state theories. These experiments do not relate to general theories of consciousness or sleep.

8.(E) Research indicates that insomnia and apnea are the most common sleep disorders, even though apnea may be very underdiagnosed.

9.(C) Marijuana is a hallucinogen. Items B and E are not categories of psychoactive drugs.

10.(D) Sleepwalking and night terrors occur during stage 4 sleep and are unrelated to dreaming and REM sleep.

11.(B) Opiates mimic the effect of endorphins in the brain, producing the pain-killing and euphoric, dreamy state associated with these drugs.

12.(C) People who use psychoactive drugs get an increased tolerance for the drugs, meaning they need more of the drugs to get the same effect. In this context, tolerance has nothing to do with the treatment or labeling of others or with sleep.

13.(D) Information-processing theory states that REM sleep and dreaming reflect the brain processing the stresses and events of our recent experience. Choice A is a definition of the activation-synthesis theory of dreams. Dreams and symbolic representations, choice B, fits Freud’s theory of dreams best. Choices C and E are incorrect distractors.

14.(E) Automatic functions like heart rate are controlled by the nonconscious level. The levels mentioned in the other choices control other parts of consciousness, except for the autonomic level, which is a created distracter and not a correct term.

15.(B) The mere-exposure effect occurs when we prefer stimuli we have seen before over novel stimuli, even if we do not consciously remember seeing the old stimuli. Priming refers to our ability to answer questions we have been exposed to before, even if we do not remember having seen the questions. Shaping is a concept in operant conditioning, primary-attribution error is a concept in social psychology that describes our tendency to attribute a person’s behavior to his or her inner disposition rather than environment. Primacy is a concept from the memory chapter.

Learning

ANSWERS TO PRACTICE QUESTIONS

1.(B) The music before a scary event in a horror movie serves as a CS. It is something we associate with a fear-inducing event (the US). In this example, preparing to be scared is the CR and the fear caused by the event in the movie is the UR. There is no such thing as a NR.

2.(C) Instinctive drift limits your pet’s gymnastic abilities. Instinctively, your dog will perform certain behaviors and will drift toward these rather than learning behaviors that go against his nature. Equipotentiality is the opposite position that asserts that any animal can be conditoned to do anything. Preparedness refers to a biological predisposition to learn some things more quickly than others. Preparedness explains why teaching a dog to fetch a stick is easier than teaching it to do a somersault. Chaining is when one has to perform a number of discrete steps in order to secure a reward. Shaping is the process one might use in teaching a dog any new trick. Shaping is when you begin by reinforcing steps preceding the desired response.

3.(C) Money is a generalized reinforcer because it can be exchanged for so many things that it is reinforcing to virtually everybody. Chocolate cake (food) and water are examples of primary reinforcers, while applause and high grades are examples of secondary reinforcers.

4.(A) In teaching your cat to jump through a hoop, continuous reinforcement would result in the most rapid learning. New behaviors are learned most quickly when they are rewarded every time. However, once the skill has been learned, partial reinforcement will make the behaviors more resistant to extinction.

5.(A) The classical conditioning training procedure in which the US is presented first is known as backward conditioning since, in most other procedures, the CS is presented first. The term aversive conditioning does not have anything to do with the order of the presentation of US and CS but rather indicates that the UR and CR are unpleasant. Simultaneous conditioning, as the name suggests, is when the CS and US are presented at the same time. Delayed conditioning is when the CS is presented first and overlaps the presentation of the US. In trace conditioning, the CS is presented first and the CS and US do not overlap. The US is then presented after a short time has elapsed.

6.(D) Tina’s slug hunting is rewarded on a variable-interval schedule. The passage of time is a key element in when she is reinforced because the slugs appear only after it rains. Since rain does not fall on a fixed schedule (for example, every third day), she is on a VI schedule. If she were on a continuous schedule, she would find slugs whenever she looked. If the slugs appeared every three days, Tina would be on a fixed-interval schedule. If she needed to turn over three rocks to find a slug, she would be on a fixed-ratio schedule. If the number of rocks she needed to turn over varied, she would be on a variable-ratio schedule.

7.(D) Exiting the elevator to avoid Lola is negatively reinforced. Your behavior, leaving the elevator, is reinforced by the removal of an aversive stimulus (Lola). Remember that reinforcement (including negative reinforcement) always increases the likelihood of a behavior as opposed to punishment, which decreases the likelihood of a behavior.

8.(C) Tolman is known for his work on latent learning, learning that occurs in the absence of a reward but remains hidden until a reward is made available. Tolman’s study involved three groups of rats running a maze under various contingencies of reinforcement.

9.(A) Many psychologists believe that children of parents who beat them are likely to beat their own children. One common explanation for this phenomenon is modeling. Modeling, or observational learning, is the idea that people or animals can learn from watching and copying the behavior of others.

10.(D) Tito’s new bed-making and chore-doing regime indicates that he is generalizing. Just as a rat will press other levers in other cages, Tito is performing more chores in an attempt to maximize his rewards. Behaviorists minimize the role of internal motivation; they believe that the environment motivates. Spontaneous recovery would be if Tito began making his bed again after his parents had stopped rewarding him and he had returned to his slovenly ways. Acquisition occurred when Tito initially learned to make his bed to earn the quarter. By assuming Tito’s parents do not reward him for making the rest of the family’s beds, he will learn ultimately to discriminate and make only his own bed.

11.(E) A rat evidencing abstract learning would learn to press a bar when shown pictures of dogs but not pictures of cats. In this example, the rat has learned the abstract concept of dog. Learning to clean and feed itself by watching its mother would be an example of modeling. Learning to associate a person’s presence with food is classical conditioning. Learning to respond only when a light is on is an example of discrimination training. Making a cognitive map of a maze without hurrying through the maze is an example of latent learning.

12.(C) B. F. Skinner believed that all learning was observable. He did not believe that learning had a cognitive component but, rather, took a radical behaviorist position that behavior was all there was. All the other choices suggest learning has a cognitive component (expectation, thought, knowing, cognition), and therefore Skinner would disagree with them.

13.(D) Charley needs to chain together a series of behaviors in order to get a reward (the bedtime story). Shaping is reinforcing approximations of a desired behavior, usually in an effort to teach it. Acquisition, in operant conditioning, is the learning of a behavior. Generalization is when one performs similar behaviors to those that will result in reinforcement. A token economy uses generalized reinforcers to control people’s behavior.

14.(A) Buying a child a video game after she throws a tantrum is an example of positively reinforcing a behavior you probably do not want (the tantrum). This example raises an important point: the word positive in positive reinforcement refers to the addition of a reinforcer and not to the goodness or badness of the act that is being reinforced. Going inside to escape a thunderstorm and getting a cavity filled at the dentist to halt a toothache are both examples of negative reinforcement (removing something unpleasant). Assigning a student detention for fighting is an example of punishment (adding something unpleasant), depriving someone of sleep is an example of omission training (removing something pleasant).

15.(B) Mr. Clayton is hoping that ignoring Lily’s behavior will extinguish the behavior. Something that is not reinforced is put onto an extinction schedule.

Cognition

ANSWERS TO PRACTICE QUESTIONS

1.(C) Functional fixedness would explain that Mr. Krohn did not think of another use for the trophy, to use it as a hammer. The representativeness heuristic is a rule of thumb for making a judgment that does not apply well to this example, retrieval is a step in the memory process, and divergent thinking is associated with creative thinking. Belief bias is our tendency to stick with a belief even when presented with contrary evidence.

2.(B) Phonemes and morphemes are elements of language. They are not used exclusively in telegraphic speech or associated with memory, the linguistic relativity hypothesis, or concepts.

3.(D) The levels of processing model would predict that you would remember the riddle because it was deeply processed. Both the levels of processing model and the three-box/information-processing model could explain the other examples, but choice D best fits levels of processing.

4.(C) Research demonstrates that the labels we apply to objects, people, and concepts affects how we think and perceive them, but there is little evidence for other ways language influences thoughts. Whorf’s hypothesis states that language restricts thought, but our cognition is not strictly limited by our vocabulary (choices A and D). The linguistic relativity hypothesis has nothing to do with language acquisition.

5.(A) The representativeness heuristic is judging a situation based on how similar the aspects are to prototypes the person holds in his or her mind. If a person has a prototype of young people as violent, she or he might use the representativeness heuristic to judge the situation. Breaking the problem down into smaller parts is a problem-solving technique. Judging a situation by a rule that is usually, but not always, true is a description of heuristics in general, not specifically the representativeness heuristic. An algorithm is a rule that guarantees the right answer. Making a judgment according to past experiences that are most easily recalled is the availability heuristic, not the representativeness heuristic.

6.(E) All five elements listed in this answer are elements in the three-box/information-processing model. Constructive memory mentioned in choice A is not part of this model (although the model can explain this phenomenon). Choice B is less complete than choice E. Choice C describes the levels of processing model. Choice D is missing long-term processing.

7.(D) The only way to determine if a memory is accurate or constructed is to look at other evidence for the “remembered” event. Brain scans and memory models cannot differentiate between true and false memories.

8.(C) Long-term potentiation strengthens neural connections by allowing them to communicate more efficiently. The other options do not describe brain processes accurately.

9.(B) The nativist theory states that we are born with a language acquisition device that enables us to learn language best as children. Choice A reflects a behavioristic view of language acquisition. Nativist theory has nothing to do with native languages or the phonics instructional method.

10.(D) Sensory memory is the split-second holding area for sensory information. Some information from sensory memory is encoded into working memory, and this process is controlled by selective attention. The three-box/information-processing model does not refer to specific brain structures like the hippocampus or the thalamus.

11.(B) By using the availability heuristic, we draw on examples that are the most readily recalled. Choice A is a good description of heuristics in general but not specifically the availability heuristic. Using a formula or rule that always gets the correct answer is an algorithm. Choice D more accurately describes the representativeness heuristic. Breaking a problem into more easily solved parts is a problem-solving technique, not the availability heuristic.

12.(A) Sensory memory holds all sensations accurately for a split second. Selective attention determines which of the memories in sensory memory we will pay attention to. Choice C is incorrect because sensory memory comes before working memory in the three-box/information-processing model. Sensory memory does not last indefinitely and does not record incorrectly, so choices D and E are incorrect.

13.(E) Recognition is matching a current experience with one already in memory. Choices A and B are incorrect descriptions of the process. The process of recall does not involve cues, and no difference in recency occurs between recalled and recognized memories.

14.(C) The critical-period hypothesis states that children need to learn language during a certain developmental period or their language may be permanently retarded. A child learning language early due to parental instruction is better evidence for the behaviorist view of language acquisition. Language-learning rates between cultures or skipping stages are irrelevant to the critical-period hypothesis. A child deprived of language early on who successfully learns language later would be evidence against the critical-period hypothesis.

15.(B) The concept of selective attention contradicts this statement. Selective attention determines what sensations we attend to and encode into short-term memory. Research shows that stimuli not attended to are not remembered, so we do not remember everything that happens to us. Sensory memory, long-term memory, and constructed memories do not obviously contradict the statement. The phenomenon of recovered memories might support the statement. Those who believe in recovered memories believe that we can remember an event for years or decades without being aware of it.

Motivation and Emotion

ANSWERS TO PRACTICE QUESTIONS

1.(D) Money is a secondary drive people learn to associate with primary drives. Choice A refers to incentive theory, choice B refers to arousal theory, and choice C refers to the hierarchy of needs. Not all psychologists agree that humans are born with instincts, so choice E is incorrect.

2.(E) The lateral part of the hypothalamus causes animals to eat when stimulated. The ventromedial hypothalamus causes animals to stop eating. The aspects described in the other answers are not controlled by these parts of the hypothalamus.

3.(B) All the other factors can be risk factors for the development of eating disorders, except for lack of willpower. As with most psychological disorders, the behaviors associated with eating disorders cannot be controlled by a sufferer through an act of will or by just trying harder. Most people suffering from an eating disorder need therapy, psychological help, and possible medication in order to stop their harmful behaviors.

4.(A) The Yerkes-Dodson law predicts that most people perform easy tasks best at high levels of arousal and difficult tasks best at low levels of arousal. The term baseline state applies best to the opponent-process theory of motivation. Self-actualization is a term used in Maslow’s hierarchy of needs, and homeostasis is a term that describes a state of equilibrium in drive-reduction theory.

5.(B) Arousal theory says humans are motivated to seek a certain level of arousal. Achievement motivation theory describes how we are motivated to meet goals and master our environment. The rest of the choices describe the theories incorrectly.

6.(A) An intrinsically motivated person motivates himself or herself with internal rewards like satisfaction. Extrinsic motivators like money are usually temporary, and the individual may lose motivation for the task when the motivator stops or does not increase. Intrinsic motivations are not easier to provide. In fact, inspiring people to become intrinsically motivated may be more difficult. Intrinsic/extrinsic motivation does not relate to Maslow’s hierarchy or primary and secondary drives. Both types of motivation are effective with a wide range of individuals.

7.(C) Theory Y managers believe workers are internally motivated. Theory X managers think workers must have external rewards in order to motivate work. Both types of managers might be equally active in work groups and might ask workers to do similar tasks. The theories might relate to types of cultures, but not the way described in choice D. Theory X and Y do not relate to optimum levels of arousal.

8.(C) Schachter said the cognitive label we apply to an event combined with our body’s reaction creates emotion. James-Lange said biological changes cause emotion, and Cannon-Bard states that emotional reactions occur at the same time as physiological changes. No theory maintains that emotions are unrelated to biological changes.

9.(C) If individuals spend an excessive amount of time in the resistance phase, it may deplete their bodies’ resources. They become more vulnerable to diseases in the exhaustion phase. The other answers do not relate to Seyle’s general adaptation syndrome.

10.(A) Studies show that if people think they are in control of an event, they report the event is less stressful. Frustration might decrease, but perceived control would not increase the feeling of frustration (choice B). Perceived control does not necessarily relate to motivation, arousal, or our heart/respiration rates.

11.(B) Homeostasis is a balanced internal state we seek by satisfying our drives. The word equilibrium does indicate balance but is not the most correct term in this context. Self-actualization is the highest need in Maslow’s hierarchy. Primary and secondary refer to drives, but the terms primary and secondary satisfaction are made-up distractions.

12.(C) The Garcia effect occurs when an organism associates nausea with food or drink through classical conditioning. This is a powerful form of learning that takes only one trial to establish. Pairing illness with food and drink is an adaptive response that may be hardwired in order to create a survival advantage. The other choices incorrectly describe the Garcia effect.

13.(D) Twin studies indicate a possible genetic influence on sexual orientation. A person whose identical twin is homosexual is more likely to be homosexual than is a member of the general population. Research indicates the other factors mentioned are not environmental factors correlated with homosexuality.

14.(C) The GAS describes different stages in reactions to stress. The other choices do not relate to Seyle’s general adaptation syndrome.

15.(D) The social readjustment rating scale is designed to measure stress. A high score on this instrument indicates the test taker experiences a high amount of stress, and this correlates with stress-related illnesses. The factors described in the other choices are not correlated with this test.

Developmental Psychology

ANSWERS TO PRACTICE QUESTIONS

1.(B) Developmental psychology can be called an applied topic because many findings from other areas are applied to the topic of maturation. Many topics such as behaviorism are easily applied to our lives. It is studied at both the graduate and undergraduate levels, many original research studies are done, and pure research is still well supported even when it involves children.

2.(D) Humans are born with reflexes that help us nurse and find our mother. We are born with all the neurons we will ever have, and we can observe babies’ behavior and infer what reflexes and abilities babies have.

3.(E) Most of our senses function at birth other than sight, which develops quickly as we mature. Lack of stimulation seems to have little effect on touch and taste, and most senses function at a normal level, not a low one.

4.(A) Teratogens are chemicals that the mother is exposed to in the environment, making them environmental influences. The rest of the answers either are not environmental in origin or do not have proven effects on a fetus.

5.(D) Motor development is not dramatically affected by parental involvement or encouragement since the rate of development is controlled mostly by development of the neurons in the cerebellum. The other answers are traits that might be greatly affected by parental involvement.

6.(C) Longitudinal studies take place over a number of years, while cross-sectional studies do not. The rest of the answers are not necessarily differences between the two types of studies.

7.(D) The monkeys in Harlow’s experiment ran to the soft mother when frightened, demonstrating the importance of a mother’s touch in attachment. The soft mothers did not feed the infant monkeys or (obviously) respond more to their needs. Stranger anxiety was present in the experiment but is not an important aspect of nurturing.

8.(E) The authoritative parenting style has been shown as the most advantageous in studies. The rest of the answers are incorrect because they identify the incorrect parenting style. Secure attachment is not a parenting style.

9.(A) The psychoanalytic theories are based on anecdotal evidence and personal inference rather than empirical research methods.

10.(B) Holden’s inability to stay in school and decide about goals indicates a search for identity, according to Erikson.

11.(D) Infants learn object permanence during the sensorimotor stage, not the preoperational or formal-operations stages. Choices C and E are not stages in Piaget’s theory of cognitive development.

12.(B) Erikson’s theory places adolescents into the identity versus role confusion stage. Adolescents would very possibly be concerned with the other factors listed in the choices, but Erikson’s theory identifies identity as the area of primary concern.

13.(D) Creating hypotheses demonstrates formal operational thought. The other choices are not stages in Piaget’s theory.

14.(C) Most of the infants Ainsworth studied demonstrated secure attachments, rather than anxious/ambivalent or avoidant attachments. The terms authoritarian and authoritative refer to parenting styles, not attachment theory.

15.(A) A schema is a mental rule we use to interpret our environment. Assimilation and accommodation are steps in the process of learning described by Piaget. We first try to assimilate new information into an existing schema, then accommodate the new information by changing the schema if we need to. A syllogism is a type of logical argument, and a hypothesis is an explanation for an environmental event.

Personality

ANSWERS TO PRACTICE QUESTIONS

1.(D) Freud described the superego as the part of the mind that acts as a conscience. The other two parts of the mind are the id and ego. The id acts according to the pleasure principle, while the ego acts as a buffer between the id and the demands of the external world. Eros is the life instinct, and the libido is the energy that drives Eros.

2.(A) A test that does not yield consistent results is not reliable. Such a test may still have been standardized and normed, both of which mean that it has been pretested on a large population and structured so that certain percentages of people answer each question in certain ways. Projective tests are used by psychoanalysts to try to see what is in a person’s unconscious. Fair is not a scientific term; tests may be perceived as unfair for a variety of reasons.

3.(B) The humanistic model of personality arose in opposition to the determinism of earlier models. One of the fundamental precepts of humanistic psychology is that people have free will, that their behavior is not predetermined. Conversely, all of the other models listed suggest that behavior is determined, at least to an extent, rather than freely chosen.

4.(D) Because the id follows the pleasure principle, it pursues immediate gratification. If Mary were all id, she would be unlikely to need many defense mechanisms; she would just do what she wanted. Similarly, such a person would rarely be in conflict over what to do and would have little interest in the proper course of action to take. A highly ethical person would be guided by her superego. A perfectionist might also have a strong superego or, alternatively, might have an anal retentive personality.

5.(C) Reciprocal determinism, also known as triadic reciprocality, is Bandura’s theory that personality arises out of the interaction of a person’s traits, environment, and behavior. All of the remaining choices are made-up distractors.

6.(E) Juan is projecting. Instead of acknowledging the feelings he has toward Sally, he views Sally as having those feelings toward him. Were Juan to displace his feelings, he would express love for someone else or something else. If Juan were to use reaction formation, he would claim to hate Sally. Juan could sublimate by directing his energies toward honing his ice hockey skills or writing poetry. Finally, were Juan to deny his crush, when asked about it, he would continue to deny it.

7.(C) Dr. Li is using the TAT (thematic apperception test). The Rorschach test asks people to look at inkblots, not people. The MMPI is a personality inventory and therefore simply involves answering questions about oneself. Factor analysis is a statistical technique, not a personality assessment. The WISC is an intelligence test.

8.(D) A lot of research suggests that shyness or inhibition is inherited. Relatively little evidence exists that suggests generosity, sense of humor, neatness, or diligence is genetically predisposed. However, many psychologists persevere in looking for this evidence.

9.(D) Self-actualization is typically defined as achieving one’s full potential; very few people ever truly self-actualize. Rather than accepting oneself as is, self-actualization is a drive to grow and improve. It is self-focused, not other focused, and does not necessarily involve the identification of core principles or bringing one’s actions into line with one’s desires.

10.(D) Honesty is not considered one of the big five personality traits. The big five are extraversion, openness, agreeableness, conscientiousness, and emotional stability.

11.(C) Feminist critics of Freud most commonly argue that, contrary to Freud’s assertion, women’s superegos are as strong as men’s. Psychoanalysts, feminist or not, generally believe in the unconscious. Feminists would be no more likely than any other group of people to argue that Freud overstressed the importance of childhood. Men and women do not seem to use categorically different defense mechanisms. While Karen Horney did suggest that men might suffer from womb envy, most feminists, including Horney, make the point that women are probably more envious of the advantages that men enjoy in society than they are of men’s penises.

12.(D) Psychoanalysts would say that Jamal has an oral fixation. They would argue that some traumatic event during the oral stage (birth to one year) caused some of his libidinal energy to become fixated in that stage. Orally controlled libido is a made-up distractor, Oedipus complex refers to boys’ supposed sexual desires for their mothers, and mother complex is a term that Jung might use.

13.(D) Rotter’s concept of locus of control has to do with how much power one feels over his or her life. Someone who has an external locus of control feels as if she or he cannot control what happens. Externals often believe their futures are in the hands of fate or luck. Someone with a positive self-concept feels good about himself or herself. Someone with a high sense of self-efficacy believes in his or her own ability to accomplish things. When we say someone has a strong libido, we usually mean she or he has a strong sex drive. Someone with a high IQ is thought to be intelligent, at least in terms of traditional measures of intelligence.

14.(E) A psychoanalyst would be most likely to use a projective test since such measures supposedly allow the person taking the test to project his or her unconscious thoughts onto the stimuli. Trait theorists might ask the person to fill out a personality inventory such as Cattell’s 16 PF. Behaviorists would monitor the person’s behavior. Social-cognitive theorists might use both self-report inventories and behavioral measures. A humanistic psychologist would also use self-report techniques and watch someone’s behavior. However, this psychologist might also want to talk to the person using an unstructured interview approach in order to get a fuller sense of the person.

15.(C) Sublimation is when one redirects unacceptable urges into a more socially acceptable pursuit. An example would be channeling your sexual frustration over your attraction to your opposite-sex parent to becoming a marathon runner. Intellectualization involves distancing oneself from the threatening issue by making it into an intellectual matter. Denial is when one denies the existence of the thought or feeling. Rationalization is when one explains away a behavior or feeling by making up a plausible excuse for it. Regression is when one returns to a behavior that was common and usually comforting at an earlier stage of life.

Testing and Individual Differences

ANSWERS TO PRACTICE QUESTIONS

1.(B) Aptitude tests aim to measure someone’s ability or potential. In this case, the test is supposed to show whether Paul has the ability to be a pilot. The test may or may not be standardized or biased. The test is not attempting to measure Paul’s intelligence. Since Paul has not yet been trained as a pilot, the test is not an achievement test.

2.(D) If a test is reliable, it yields consistent results. Standardized tests are generally given in the same way every time. A test is valid if it measures what it is supposed to measure. While valid tests are reliable, reliable tests are not necessarily valid. Whether a test is fair or biased can be evaluated in several ways as explained in the chapter. However, the fairness of a test is not synonymous with its consistency.

3.(C) The standardization sample represents the population for whom the test was intended and is used to construct the test. None of the other choices are referred to by any specific terminology.

4.(A) Practical intelligence is part of Sternberg’s triarchic theory of intelligence. Gardner’s multiple intelligences include linguistic, spatial, logical-mathematical, musical, bodily-kinesthetic, interpersonal, intrapersonal, and naturalist.

5.(C) Mrs. Cho is concerned about the content validity of her test. A test that fairly represents all the material taught in her class has content validity. Validity, in general, measures how well a test measures what it is supposed to measure. In order for Mrs. Cho’s test to have construct validity, we would need to know that the test was successful in differentiating between varying levels of achievement in Mrs. Cho’s class. If the test has criterion validity, we would have to know that the test successfully identified either those students who had excelled in their study of American literature (concurrent validity) or those students who would excel in the future (predictive validity). Reliability is a measure of how consistent the scores are on a test. Test-retest reliability involves giving the same test to the same population on at least two different occasions and measuring the correlation between the sets of scores. Split-half reliability is when one test is divided into two parts and the correlation between people’s scores on the two halves is measured.

6.(C) The WISC (Wechsler intelligence scale for children) yields a deviation IQ score. The mean on the WISC is set at 100. Therefore, someone who scores 100 has scored at the 50th percentile on the test. The standard deviation on the WISC is set at 15. Since approximately 34 percent of the scores in a normal distribution fall between the mean and one standard deviation above the mean, Astor’s score at the 84th percentile indicates that he scored almost exactly one standard deviation above the mean. Therefore, to compute Astor’s score, we simply have to add the mean (100) to one standard deviation (15).

7.(C) Spearman argued that intelligence could be boiled down to one ability known as g. The g stands for general intelligence. Spearman also discussed s, which stands for specific intelligences. The other letters are all simply distractors.

8.(B) The strongest evidence presented for intelligence to be highly heritable is that monozygotic twins separated at birth have extremely similar IQ scores. Monozygotic twins share 100 percent of their genetic material. If they are separated at birth and therefore raised in different environments, similarity in their IQ scores argues for the influence of nature or heritability. Parents’ IQ scores do tend to correlate positively with those of their children, but this similarity could be explained by either genetic or environmental factors. Dizygotic twins and other siblings share the same amount of genetic material on average (50 percent). Therefore, if the former score more similarly on IQ tests, an environmental influence is suggested. For instance, dizygotic twins may be treated more similarly than other siblings and grow up during the same time period. Since adopted children do not share any genetic material with the parents who adopted them, similarities must be due to environmental factors. Differences in average IQ scores between ethnic groups could be explained by either genetic or environmental factors.

9.(A) Vocabulary tends to increase with age and does not depend on speed; such attributes characterize crystallized intelligence. Choices B, C, and D are more typically associated with fluid intelligence because they involve speed and learning new things. Choice E is also more likely to be associated with fluid intelligence because the ability to perform complex mathematical calculations in one’s head is linked to working memory that tends to decline with age.

10.(D) Power tests consist of items of varying levels of difficulty because their purpose is to identify the upper limit of a person’s ability. Speed tests are given in a small amount of time since they seek to test how quickly someone can solve problems. Power tests could be given individually or in a group. Having a pure measure of achievement is impossible. IQ tests yield IQ scores.

11.(C) People with high EQs would be likely to find jobs well suited to their individual strengths. Emotional intelligence is thought to help people achieve what they want to achieve. Someone who has a high EQ will not necessarily want a high-paying job, go to college, be a creative problem solver, or have many close friends.

12.(D) Since the personality test does not seem to have resulted in an accurate depiction of Mary’s personality, the test lacks validity. If repeated administrations of the test yielded similar results, the test could still be reliable. The test may or may not have been standardized. Consistency is generally equated with reliability. If the test lacks validity, it will not have practical worth, but the latter is not a psychological term.

13.(D) Scores on the Stanford-Binet IQ test are computed by dividing mental age by chronological age and multiplying by 100. Since 10 divided by 8 equals 1.25, Santos has an IQ of 125.

14.(A) The Flynn effect is the finding that intelligence seems to be increasing with every generation. Although television is often cast as a great social evil that rots the minds of our nation’s youth, one hypothesized contribution to the Flynn effect is the exposure to the complex and rapid visual stimuli that appear on television. Linguistic skills do not decline with age. The statements in choices D and E are true but are not known as the Flynn effect.

15.(D) Adherents to a nature perspective often emphasize the effect of genetic makeup in shaping personality. All the other factors (environment, learning, reinforcement, and culture) are associated with a nurture perspective.

Abnormal Psychology

ANSWERS TO PRACTICE QUESTIONS

1.(E) Panic disorder is an anxiety disorder. Although both obsessive-compulsive disorder and post-traumatic stress disorder were considered anxiety disorders under the DSM-IV-TR, both have been recategorized. Conversion disorder is a somatic symptom disorder, and mania is a mood disorder.

2.(D) Distortion of thought is the characteristic common to all types of schizophrenia. Schizophrenia is distinct from both depression and dissociative identity disorder, although, as discussed in the chapter, it is often confused with the latter. Although some schizophrenics manifest flat affect, it is not a symptom shared by all schizophrenics. Only some schizophrenics experience delusions of persecution.

3.(D) Perceiving sensory stimulation when none exists defines a hallucination. Delusions are irrational thoughts but do not involve a belief in the existence of sensory stimulation. Obsessions are persistent, unwanted thoughts. Compulsions are unwanted, repetitive actions that people engage in to reduce anxiety.

4.(C) Linda is suffering from dissociative identity disorder, a type of dissociative disorder.

5.(A) The DSM mainly contains the symptoms of the various disorders. For many disorders, neither causes nor treatments are addressed in the DSM because beliefs about both causes and treatments often depend on the theoretical model to which one subscribes.

6.(A) A double bind is when someone is told to do something and then punished for doing it. While some researchers theorize that double binds cause schizophrenia, they would be an environmental cause, not a physical or medical cause. All the other choices are biological factors that theoretically play a role in schizophrenia.

7.(D) Psychologists who draw on various perspectives in their work are known as eclectic. The other terms have no specific psychological meaning.

8.(C) Depression is associated with low levels of serotonin.

9.(E) Narcissism is the love of oneself. People who view themselves as the focus of the world would most likely be classified as having narcissistic personality disorder.

10.(B) Psychoanalysts view depression as the result of anger turned inward. Biomedical psychologists would see depression as the result of some biological cause. Cognitive psychologists would locate the cause in the person’s style of thinking. Behaviorists believe depression is caused by one’s reinforcement history. Psychologists adhering to the sociocultural model would fault aspects of society such as racism or poverty.

11.(E) Sociocultural psychologists believe that mental illness is mainly caused by certain negative aspects of society such as sexism.

12.(D) Schizophrenia is associated with high levels of dopamine and Parkinson’s disease with low levels of dopamine. Schizophrenia is often treated with antipsychotic drugs. Excessive use of amphetamines is associated with both high levels of dopamine and schizophrenic-like symptoms.

13.(A) Anand’s symptoms suggest he has conversion disorder, a type of somatoform or somatic symptom disorder in which no physical cause can be found for a physical complaint. The hallmark of dissociative amnesia is difficulty remembering things that are potentially disturbing and which cannot be explained by a physical trauma. GAD is generalized anxiety disorder, a constant, low-level sense of nervous tension. SAD, seasonal affective disorder, is a type of mood disorder in which people become depressed during prolonged periods of bad weather. OCD, obsessive-compulsive disorder, is an anxiety disorder in which unwanted, persistent thoughts push people to perform unwanted, repetitive actions to reduce anxiety.

14.(E) Paraphilias involve sexual arousal and interest in people, objects, or situations not generally considered arousing. More specifically, Reni has a fetish since he is aroused by an object. Pedophilia is a sexual attraction to children. Masochism is when one needs to be hurt in order to be sexually aroused, and sadism is when one is aroused by inflicting pain on others. Exhibitionism is when one is aroused by exposing him- or herself to others.

15.(E) Bulimics tend to be overly concerned with their weight and body image. These concerns lead them to engage in the binge-purge cycle that typifies bulimia. A growing number of men suffer from bulimia. Not all bulimics use vomiting to purge unwanted calories. Other methods include using laxatives, diuretics, and excessive exercise. Anorexics, not bulimics, lose more than 15 percent of their normal body weight. As is true of most, if not all, disorders, the cause of bulimia is a matter of some debate.

Treatment of Psychological Disorders

ANSWERS TO PRACTICE QUESTIONS

1.(D) Psychoanalysts see the root of disorders in unconscious conflicts. Therefore, their initial focus is to bring the conflict into conscious awareness. Due to patients’ defenses, psychoanalysts need to employ special techniques to reveal the contents of the unconscious. Dream analysis is one such technique. Behaviorists are interested only in the clients’ behavior. Cognitive therapists are more likely to explore the clients’ waking thoughts. Humanistic psychologists will try to help clients clarify their own thoughts and feel positively about themselves. Therapists with a biomedical orientation will be most likely to recommend somatic therapies, like drugs.

2.(E) Coretta’s therapist is nondirective and therefore is most likely to have a humanistic orientation. An example of such a therapy is Carl Rogers’ client-centered therapy.

3.(B) Psychoanalysts believe that disorders are caused by unconscious conflicts that are not immediately apparent. Treating the symptom is essentially pointless since it may disappear but a new one will take its place, a phenomenon known as symptom substitution. Therefore, when Craig went to a behaviorist and was cured of his test anxiety in a few months, psychoanalysts would predict the development of a new symptom. Free association is a technique used by psychoanalysts to uncover the contents of the unconscious. An anxiety hierarchy is part of systematic desensitization, a behaviorist treatment for anxiety disorders. While no such thing as problem transference exists, transference occurs when patients put feelings about significant people in their lives onto the analyst. Analysts necessarily engage in interpretation to figure out the source of their patients’ difficulties.

4.(D) Both systematic desensitization and in vivo desensitization are treatments for phobias and other anxiety disorders. In systematic desensitization, clients imagine the different levels of the anxiety hierarchy. However, in vivo densensitization involves experiencing the anxiety-provoking situations. Similarly, implosive therapy involves imagining an intensely feared situation until the fear is extinguished, while flooding consists of experiencing the highly anxiety-provoking situation until the fear is extinguished.

5.(C) Cognitive therapy is often somewhat confrontational as cognitive psychologists attempt to change the irrational ways their clients think. A common problem that cognitive psychologists fight is depressed people’s tendency to globalize and internalize negative thoughts as evidenced by Ivan’s assertion that he is completely worthless. The statements in choices A and E would more likely be made by a psychoanalytic or psychodynamic theorist as they suggest that the roots of many problems are laid in childhood and they stress the importance of the unconscious mind to which dreams are clues. The statement in choice B would most likely be made by a behaviorist as it stresses the importance of behavior, and the statement in D reflects humanistic psychologists’ belief in being nondirective and reflecting back what their clients say.

6.(E) Counterconditioning involves replacing a CR with a new CR. In systematic desensitization, clients are taught to replace fear with relaxation. None of the other therapies listed are based on learning principles.

7.(A) Somatic therapies, as opposed to psychotherapies, view the cause of the problem in biology and therefore involve medical treatments. MAO inhibitors are drugs sometimes prescribed to treat depression. All the other choices are types or aspects of psychotherapy.

8.(A) Classical conditioning is a kind of learning that results from associating two things, one of which is an unconditioned stimulus, together. In operant conditioning, the consequences of one’s actions lead to learning. Token economies are based on the principles of operant conditioning; people will act in certain ways to attain rewards. Implosive therapy, flooding, and systematic desensitization are all based on classical conditioning methods. Aversion therapy is a broader term that includes both classical and operant conditioning methods.

9.(B) Transference is when patients misdirect feelings toward important people in their lives onto the therapist. Resistance also commonly occurs in psychoanalysis but is when a patient rejects the analyst’s interpretations or otherwise seeks to thwart the therapeutic process.

10.(C) Modeling consists of observation and imitation. Jeb watches someone model how to ask for a raise, and then he practices that skill himself.

11.(C) Psychoanalysis stresses the importance of early childhood experience. Psychoanalysts spend a lot of time exploring patients’ early lives. Cognitive therapists focus on helping their clients deal with the present. Neither type of therapist is particularly reticent; humanistic therapists are. Neither psychoanalysts nor cognitive therapists emphasize the importance of behavior; that focus characterizes behaviorists. Psychoanalysts, not cognitive psychologists, do see repressed thoughts from childhood as the root of most adult problems and do not face their patients.

12.(A) Eclectic therapies incorporate aspects of several different models rather than strictly adhere to one theoretical orientation. Psychodynamic therapy, while based on psychoanalysis, tends to incorporate aspects of other models as well. Client-centered therapy is humanistic. Aversive conditioning and token economies are behavioral. Psychoanalytic therapy is, of course, psychoanalytic.

13.(D) Haldol is an antipsychotic drug. Prozac is used to treat depression, lithium to treat mania, and Miltown and Valium to treat anxiety disorders.

14.(B) Memory loss, although often temporary, is a common side effect of ECT. Tardive dyskinesia is a side effect of the antipsychotic medications used to treat schizophrenia.

15.(A) The deinstitutionalization movement occurred when many patients were released from mental hospitals in the 1960s and 1970s. Many were schizophrenics who, unable to find jobs and adequate care outside of the hospital setting, became homeless.

Social Psychology

ANSWERS TO PRACTICE QUESTIONS

1.(C) A task that requires groups to cooperate is an example of a superordinate goal. Such superordinate goals are effective in breaking down hostility between groups. Contact between antagonistic groups without superordinate goals is less successful, and simply avoiding members of the other group is unlikely to decrease the intergroup hostility. While guest speakers may be able to influence the group members’ attitudes, they will be less effective than the use of superordinate goals. Punishing the groups may actually increase the antagonism between them.

2.(E) Tanya made a large request and, when refused, came back with a smaller, more reasonable sounding request. This compliance strategy is known as door-in-the-face. Foot-in-the-door is when one makes a small request and, once that request is agreed to, follows up with a larger request. Had Tanya brought her teacher an apple and then made her request she would have been attempting to capitalize on norms of reciprocity, the idea that one good turn deserves another. Although Tanya is, in fact, attempting to broker a compromise and engage in some bargaining, the strategy she used has a more specific, psychological name.

3.(A) The dependent measure in the Milgram experiment was the level of shock the participants thought they were administering. While Milgram manipulated a number of independent variables including the location of the learner relative to the teacher, the dependent variable he measured was always how far the participants would go in shocking the learners. The length of the line is a reference to the Asch conformity experiment. No groups were involved in the Milgram experiment. The experimenter’s instructions did not vary between conditions.

4.(B) Pluralistic ignorance is defined as the tendency of people to look toward others for cues about how to act, particularly in emergency situations. Pluralistic ignorance is often tested in bystander intervention studies. Pluralistic ignorance can be seen as a kind of modeling or conformity that occurs in emergencies. However, pluralistic ignorance is a superior answer due to its clear relationship to emergency situations. Diffusion of responsibility is the finding that the more people who witness an emergency, the less likely any one is to intervene.

5.(E) Communications are made more effective when communicators are attractive, famous, and/or perceived as experts. All of these factors enhance the persuasiveness of an appeal.

6.(D) Your failure to consider the role of situational factors in explaining your new neighbor’s knowledge of ancient Greece is known as the fundamental attribution error. The self-fulfilling prophecy effect is when one person’s expectations affect another person’s behavior. Pluralistic ignorance is the tendency to look to others for hints about how one is supposed to act in certain situations. Confirmation bias is the tendency to focus on information that supports one’s initial ideas. Cognitive dissonance is the tension felt when one holds two contradictory ideas.

7.(E) In Asch’s conformity study, approximately 70 percent of participants gave at least one incorrect answer.

8.(A) Janine is experiencing cognitive dissonance. The combination of her hatred of the color orange and her ownership of a lot of orange clothing results in a tension called cognitive dissonance. It will motivate her to reduce the tension by either changing her opinion of orange or radically altering her wardrobe. All of the other terms are made up.

9.(D) When the presence of other people inhibits someone’s performance, social impairment, the opposite of social facilitation, has occurred. Social loafing is the tendency of people to exert less effort in a group than they would if they were alone. Groupthink is the idea that because group members are often loathe to express opinions different from those of the majority, some groups fall prey to poor decisions. Deindividuation is when people in a group lose their self-restraint due to arousal and anonymity. Diffusion of responsibility is one way to explain the inverse relationship between group size and the expression of prosocial behavior.

10.(B) According to Kelley, people use information about consensus, consistency, and distinctiveness when analyzing the cause of people’s actions. The other terms, while related to psychology, are not generally important in attribution theory.

11.(B) The students in the school are evidencing self-serving bias, the tendency to take more credit for good outcomes than bad ones. When the football team wins, they want to identify with them and therefore say “We are awesome.” When that same team loses, the students distance themselves from the players, explaining that “They were terrible.” Fundamental attribution error is a different attributional bias. It explains that people overestimate the role of personal factors when explaining other people’s behavior. The self-fulfilling prophecy effect is the finding that people’s expectations about others can influence the behavior of those others. The false consensus effect is another example of an attributional bias. It says that people overestimate the number of people who share their beliefs. Finally, conformity is the tendency for people to go along with a group.

12.(B) A prejudice is an attitude, while discrimination involves an action. Santiago has a negative attitude or prejudice toward cheerleaders. Billy and Mr. Tamp are engaging in discrimination by acting differently toward different groups of people. Athena may be really fast or overconfident, but she is not evidencing a prejudice. Ginny’s belief that all Asians are smart is a stereotype that may or may not lead her to have some kind of prejudice against Asians.

13.(A) Self-disclosure is the process by which two people become closer by sharing intimate details about themselves. Deindividuation is when people in a group lose their self-restraint due to arousal and anonymity. In-group bias is the preference that people show for members of their own groups. Dual sharing is a made-up term, and open communication, while healthy in a relationship, does not describe this specific exchange.

14.(C) By dividing his students into groups, Mr. Simpson fostered the development of in-group and out-group bias, the belief that members of one’s own group are superior to members of other groups. While Jody’s aggressive behavior cannot be fully explained by Mr. Simpson’s grouping, the fact that he attacks only members of other groups suggests that out-group bias may play a role. Group polarization is the tendency of groups to take more extreme positions than those taken by their individual members. Since Jody acts alone and not as part of a group, his aggression cannot be seen as an example of deindividuation. Superordinate goals are helpful in reducing conflict between groups by making their success contingent upon their cooperation. Groupthink is the idea that because group members often avoid expressing opinions different from those of the majority, some groups fall prey to poor decisions.

15.(A) Rosenthal and Jacobson’s “Pygmalion in the Classroom” study illustrated the self-fulfilling prophecy effect, the ability for people’s expectations about others to influence how those others behave. Attitudes do not always predict behavior well, as LaPiere’s study evidenced. Contact is not usually sufficient to break down prejudice. The just-world belief tells us that people like to think that others get what they deserve. Cohesive groups sometimes engage in groupthink, resulting in bad decisions.

Practice Test 1

ANSWERS EXPLAINED

1.(D) Myelin is a fatty tissue that surrounds the axons of some neurons and helps speed the movement of the action potential (essentially an electric charge) down the neuron. Neurotransmitters are chemicals that are stored in the terminal buttons and ultimately released into the synapse. Hormones are part of the endocrine system and are not involved in neural transmission.

2.(A) The “c” in the word cat is best described as a phoneme. Phonemes are the smallest units of sound in a language. Morphemes are the smallest units of meaning in a language; the entire word cat is a morpheme as it cannot be broken down into smaller meaningful units. Holphrases are single words that toddlers use to express more complex ideas as they first learn language. A toddler might say “cat,” meaning “look, there’s a cat” or “I’d like to pet that cat.” Like phonemes, syllables are units of sound, but they are often composed of multiple phonemes; for instance, the word cat contains three phonemes but only one syllable. A grapheme is a written symbol for a sound in a language; letters are graphemes but so are the letter combinations that make up multiletter phonemes, for instance, “ph.”

3.(E) When first born, humans’ dominant sense is hearing. In fact, research shows that fetuses are able to hear in the womb. By the age of about 6 months, sight replaces hearing as a typical baby’s dominant sense.

4.(A) Ryan is most likely to be classified as suffering from GAD, generalized anxiety disorder. As described in the question, GAD involves a nearly constant low-level arousal of the autonomic nervous system. Major depressive disorder is a mood disorder rather than an anxiety disorder. Hypochondriasis is a type of somatoform disorder in which sufferers fear that typical body sensations are indicative of medical problems. People who have obsessive-compulsive personality disorder tend to be inflexible and overly concerned with things like neatness, routine, and perfection. PTSD (posttraumatic stress disorder) follows some kind of trauma and is usually marked by anxiety caused by memories, flashbacks, and nightmares having to do with the trauma.

5.(E) The psychoanalytic perspective was popular at the turn of the twentieth century in Western Europe. The neuroscience, evolutionary, and cognitive perspectives had yet to capture most people’s interest, largely because of the lack of technological resources to study people from these perspectives. While the behaviorist perspective was beginning to gain followers in the United States during this time, Western Europeans remained more interested in Freud’s psychoanalytic approach.

6.(C) Pascale would best be classified as a cognitive psychologist due to his interest in children’s processing strategies. Sociocultural psychologists focus on the role of various societal factors in shaping people. Clinical psychologists treat clients or research psychological disorders or therapies. Behaviorists would reject the importance of mentalistic concepts like “processing” strategies. Personality psychologists focus on questions concerning the development of people’s personalities.

7.(E) Cyan’s test would be classified as an aptitude test because it is intended to show a person’s potential to be a good civil rights attorney. A speed test is comprised of many items and is meant to demonstrate how quickly a person can answer questions. An achievement test measures people’s knowledge of a given area. An EQ test would measure emotional intelligence, whereas an IQ test would measure intelligence.

8.(A) The learning process described would best be labeled classical conditioning because Marva’s children have come to associate the sound of a car door slamming and their mother’s screaming. Instrumental learning and operant conditioning are similar in that they involve learning to pair a consequence with a behavior. Observational learning, or modeling, is learning via observation and imitation. Latent learning is learning that is not evidenced until a reward is offered.

9.(B) The fact that the children now cringe when they hear any car door slam is an example of generalization since the children have generalized their response (CR) to the sound of their mother’s car door slamming to the sound of all car doors slamming. Discrimination would be the opposite phenomenon—if the children learned to cringe only to the sound of their mother’s car door and not other car doors. Acquisition in this example is when the children began to cringe to the sound of their mother’s slamming car door. Extinction is the opposite of acquisition—if Marva learned to manage her anger and stop screaming at her children, they would eventually unlearn the cringing response to the sound of her car door. Spontaneous recovery would be if, after extinguishing the cringe response, the children at a later date cringed again upon hearing a slamming car door.

10.(C) Your friend’s comment is a product of hindsight bias, also known as the “I knew it all along” phenomenon. Overconfidence is people’s tendency to believe they have performed better than they have. A theory is a statement that seeks to explain some set of observations and is used to generate hypotheses. A hypothesis is a testable prediction. Response bias refers to any one of a host of factors that cause people to answer questions in ways that do not reflect their actual opinions or beliefs.

11.(C) A low level of glucose in Sam’s bloodstream is most likely to make him want to eat a meal. Low blood sugar levels are an important cue that one is hungry. None of the other choices are tied directly to blood sugar rates.

12.(E) A trait theorist would be most interested in the results of the MMPI. The MMPI (Minnesota Multiphasic Personality Inventory) is a self-report instrument. Trait theorists use such inventories to help describe personality. Psychoanalytic theorists would not put much stock in people’s conscious reports of their own traits since psychoanalysts believe important motivations are largely inaccessible in the patient’s unconscious. Humanistic psychologists would prefer a more in-depth interview/conversation with a client in order to understand fully the unique features of each person. Behaviorists are far less interested in what people say than what they do. Finally, biological theorists are more interested in physiological factors than one’s self-reported characteristics.

13.(C) Light enters the eye through an opening in the iris. The iris, the colored part of the eye, is a muscle that controls the size of the pupil, the hole through which light enters the eye. After passing through the pupil, the light is refracted by the lens and ultimately lands on the retina at the back of the eye. The blind spot is the part of the retina where the axons of the ganglion cells exit the eye en route to the brain; it is known as the blind spot because there are no photoreceptors there. The fovea is the part of the retina in which you have highest visual acuity due to the lack of summation there.

14.(D) In people, rods, unlike cones, have a lower absolute threshold for light; that is, rods function in conditions of dimmer light than do cones. Rods are located on the periphery of the retina, do not respond to color, and are good at detecting motion. Both rods and cones synapse with bipolar cells.

15.(D) The term group polarization describes the phenomenon of groups taking more extreme positions than the individuals that comprise the group. Social facilitation is the finding that the presence of others improves performance on well-learned tasks. Deindividuation is the loss of self-restraint that occurs in groups under conditions of high arousal and relative anonymity. Groupthink is the tendency of groups to suppress dissent and make bad decisions. Conformity is when one alters one’s attitudes or behaviors to go along with a group.

16.(B) The siblings have different circadian rhythms, approximately 24-hour cycles in biological and behavioral processes including those that regulate alertness, body temperature, and heart rate. Sleep cycles are an example of an ultradian rhythm, a cycle that happens multiple times a day. The sleep cycle is approximately 90 minutes long, and we cycle through it several times over the course of a night’s sleep. Even though the siblings may differ in terms of their daily activities and personalities, these are unlikely to account for their consistent differences in peak alertness time.

17.(D) The feeding tubes represent a somatic treatment, the discussion of how her beliefs are irrational involves a cognitive approach, and the belief that her early family relationships are important belies a psychoanalytic bent. The combination of these three different perspectives signals that an eclectic approach is being used.

18.(B) Counterconditioning lies at the heart of systematic desensitization. Counterconditioning is a type of classical conditioning that involves replacing one’s initial reaction to a stimulus with a new reaction. Systematic desensitization is often used to treat phobias by replacing the fear response to the stimulus with a relaxation response. Flooding is based on extinction. REBT and stress inoculation are cognitive therapies, and token economies make use of operant, as opposed to classical, conditioning principles.

19.(C) Humanistic personality theory is criticized for having an unrealistically optimistic view of human nature. Humanistic psychologists generally believe that all people are innately good and motivated to fulfill their own unique potential, ideas that some critique as overly optimistic. None of the other perspectives listed have quite as positive a view of human nature.

20.(D) Superordinate goals have been found to decrease prejudice, perhaps most notably in Sherif’s camp study and Aronson’s use of the jigsaw classroom. All the other phenomena help contribute to prejudice. Stereotypes are specific ideas about a group applied to all its members. Scapegoating is the practice of blaming members of a disliked group for one’s own misfortunes; it can serve to rationalize one’s prejudice. Out-group homogeneity is the belief that members of one’s out-groups are essentially all the same, thus facilitating prejudice. Discrimination is acting on one’s prejudices; through the process of cognitive dissonance, discrimination may serve to amplify one’s prejudices.

21.(D) One of the hallmarks of the experimental method is the ability to control for various confounding variables, while naturalistic observation is primarily a descriptive method that tried to convey a realistic picture of a phenomenon rather than artificially manipulate and control it. Experiments typically yield quantitative data; naturalistic observation often yields both quantitative and qualitative data. Naturalistic observation involves observation, not surveys; in experiments, the dependent variable measured may or may not be behavioral. While naturalistic observation typically takes place in something we could describe as the field, experiments may take place in either the lab or the field. Finally, both experiments and naturalistic observation may yield statistically significant results.

22.(D) According to humanistic psychologists, psychological disorders are most likely caused by unfilled needs. Humanistic psychologists believe that people are naturally good and that as long as their needs are met, they will develop into healthy individuals. However, if their needs (physical, emotional, etc.) are not met, psychological disturbances may result. Psychologists adhering to the biological perspective would be most likely to name neurochemical imbalances as the cause of psychological disorders. An unhealthy attributional style is a cognitive explanation. The phrase “learned associations” implies a behaviorist perspective, and the phrase “genetic predispositions” suggests an evolutionary approach.

23.(B) A fetus with the genotype XXY will most likely suffer from sterility as an adult. The XXY genotype is known as Klinefelter’s syndrome and is one of the few chromosomal abnormalities that does not typically result in a miscarriage. Men with Klinefelter’s syndrome are somewhat less likely to be color blind than other men since the trait for color blindness is recessive and carried on the X chromosome; therefore, these men would have to have a copy of the gene on each of the X chromosomes to express it. Depression is not associated with Klinefelter’s syndrome, and everyone who has Klinefelter’s syndrome is a male due to the presence of the Y chromosome.

24.(E) Most children will develop the ability to clap their hands before any of the other skills listed. Typically, infants learn to clap hands in the second half of their first year. In general, gross motor skills (e.g., clapping) tend to develop before fine motor skills (for example, writing, cutting with a knife, turning pages of a book). Most children do not speak in anything resembling full sentences until they are well over 2 years old.

25.(D) Antonia has a cat. The first time she sees a rabbit, she calls it a cat. Her error results from the process of assimilation. Assimilation, as defined by Piaget, is the ability to take in new information using one’s existing schemas. Antonia had a schema for a cat and used it to make sense of a new animal, a rabbit. Accommodation will occur if Antonia is corrected and told that this new animal with longer ears and a shorter tail is a rabbit, and she then creates a new schema for rabbits. Discrimination and generalization are terms used together in discussing learning. Discrimination is when one can tell the difference between a stimulus and similar stimuli, and generalization is when one responds the same way to similar stimuli as one did to the stimulus with which one was originally trained. Habituation is a decrease in response to a repeated stimulus.

26.(C) Divergent thinking is a term closely associated with creativity. Convergent thinking involves looking for a single correct answer or using one prescribed method of problem solving; divergent thinking typically involves more open-ended problems. Hypothesis testing can absolutely have creative elements but is not synonymous with creativity as is divergent thinking. Contextual intelligence is a kind of practical intelligence described by Robert Sternberg. Habituation is a decrease in response to a repeated stimulus. Echoic memory is the auditory portion of sensory memory that lasts a few seconds.

27.(B) The somatic nervous system is part of the peripheral nervous system. The peripheral nervous system is comprised of the somatic (or skeletal) nervous system and the autonomic nervous system and collectively contains all parts of the nervous system outside of the central nervous system. The central nervous system contains the brain and spinal cord. The autonomic nervous system has two parts: the parasympathetic and sympathetic nervous systems.

28.(B) As animals evolved and became more advanced, their brains typically became larger, relative to the size of their bodies. However, the part of the brain that has increased the most in size and a part that is not even present in many animals is the cerebral cortex. This change makes sense given that the cortex is the seat of high-level thought. The pons and reticular formation are both part of the brainstem, an older part of the brain. The forebrain includes the cortex as well as less advanced structures like the hypothalamus.

29.(E) Of the disorders listed, major depressive disorder would respond best to cognitive behavioral therapy. Cognitive behavioral therapy (CBT) is a type of talk therapy in which the clinician tries to reason with the client. Given the disorganized thought and difficulties with oral expression of schizophrenics, they are poor candidates for such an approach. Dissociative identity disorder (DID) might also be difficult to treat from this perspective because cognitive psychologists focus on conscious thought and might be skeptical about the unconscious and the existence of DID. Finally, while CBT might help someone with bipolar disorder while in the depressed phase, it would probably be less effective in dealing with people in the manic phase as they are typically not thinking logically nor receptive to rational arguments.

30.(B) The basilar membrane is found in the inner ear. The basilar membrane is in the cochlea, and the hair cells are embedded in this membrane. The hammer is one of the ossicles in the middle ear. The tympanic membrane, or eardrum, divides the outer and middle ears. The pinna and auditory canal make up the outer ear.

31.(B) An evolutionary psychologist or sociobiologist would assert that a key motivation for people is the desire to spread their genetic material. Cognitive psychologists believe that thoughts are at the core of motivation. Humanistic psychologists assert that people are motivated to fulfill their needs. Psychodynamic psychologists believe unconscious thoughts are important motivators. Behaviorists believe we are motivated by the contingencies of reinforcement to which we have been exposed.

32.(B) Psychodynamic theorists would say that Max is sublimating. Sublimation is a Freudian defense mechanism that involves taking inappropriate emotions (for example, fury) or desires and redirecting them toward more socially acceptable behaviors (for example, artwork). The other choices are all other defense mechanisms. If Max used displacement, he would take his anger out on a less threatening target than Dylan. If Max used rationalization, he might think to himself that he can get a better prom date. If Max repressed his fury, he would basically forget about the event by pushing the memories into his unconscious. Finally, if Max used projection, he might claim that Dylan and/or Suzy were angry with him when, in fact, it is he who is angry.

33.(A) The relationship between arousal and performance is known as the Yerkes-Dodson law and is expressed by a graph that looks like an inverted “U”. As arousal increases, so does performance—up to a point. When arousal becomes too high, performance suffers.

34.(A) Measures of variability (e.g., variance, standard deviation, range) show how spread out a distribution of scores is. The range (the highest score minus the lowest score) is less informative than the variance or standard deviation, which shows the average distance of all scores in a distribution from the mean. The mean, median, and mode are measures of central tendency; their aim is to mark the center of a distribution, not to say how spread out the distribution is.

35.(E) Since Coach Peterson takes away playing time that Jake usually has, the technique described is omission training. An example of positive punishment would have been if the coach had Jake run laps (added something unpleasant). Neither negative reinforcement nor positive reinforcement would have been a good tactic for the coach to use on Jake’s lateness, since, by definition, they would have encouraged Jake to be late again. However, the coach could reinforce Jake’s future appropriate behaviors toward the same end. He could praise Jake when he arrives on time (positive reinforcement) or excuse Jake from the laps run at the end of practice when he is on time (negative reinforcement). Modeling is learning via observation and imitation; the coach could model being on time himself.

36.(D) Gardner, Sternberg, and a host of other contemporary intelligence researchers believe that schools focus too much on traditional subjects and methods. Gardner has proposed that people have multiple abilities that ought to be considered intelligences, including bodily-kinesthetic intelligence, musical intelligence, and interpersonal intelligence. Sternberg believes schools should encourage students’ creative and practical abilities as well as their analytical ones.

37.(B) Delusions of grandeur are most characteristic of paranoid schizophrenia. An example of a delusion of grandeur would be the belief that one was a god or a king. Along with delusions of persecution, delusions of grandeur are amongst the most common symptoms of paranoid schizophrenia.

38.(D) Faye believes that victims of natural disasters are foolish because they didn’t develop better advance detection and warning systems. Faye is manifesting the just-world bias, the belief that because the world is a fair place, good things happen to good people and bad things happen to bad people. The false consensus effect is our tendency to overestimate the number of people whose views are similar to our own. The self-fulfilling prophecy effect is the ability of one person’s expectations to elicit behavioral confirmation in another. Self-serving bias is the tendency to take more credit for good outcomes and less credit for bad outcomes than one deserves. The Barnum effect is people’s willingness to see themselves in vague, stock descriptions.

39.(A) Four-year-old Kate positions herself squarely in front of all the other kids to watch a magician. Piaget would attribute this to Kate’s egocentrism. Piaget would say that Kate does not have the cognitive capacity to realize that she is blocking others’ view; she is only capable of seeing things from her own perspective. Piaget believed infants develop object permanence, the understanding that an object still exists even when no longer in view, toward the end of the first year. According to Piaget, children learn to conserve around age 7, when they enter concrete operations. Conservation is the knowledge that a change in the form of matter does not change the amount of matter that exists. Animism and artificialism are limitations in the thought of the preoperational child. Animism is when one attributes life or consciousness to inanimate objects, and artificialism is the belief that everything has been created by people.

40.(E) The inability to encode new memories is known as anterograde amnesia. The prefix ante means “before,” and the “ante” in anterograde indicates that the trauma causing the memory problem preceded the memories with which it is interfering.

41.(B) The brainstem is comprised of the hindbrain and midbrain. The brainstem houses some of the most basic functions of the body and is present in similar form in less complex animals, such as reptiles.

42.(B) The limbic system plays an important role in regulating emotion. The amygdala is part of the limbic system and is vital in terms of regulating emotion. Maintaining balance is associated more with the cerebellum. Monitoring arousal is the job of the reticular activating system, also known as the reticular formation. The thalamus is important in the initial processing of most sensory information, and the prefrontal cortex is the part of the brain most associated with planning for the future.

43.(A) Loftus’s work shows that memory is reconstructed; that is, a memory is not like an exact replica of what happened but rather is built each time a person thinks about it and is therefore affected by all sorts of extraneous information. The serial position effect refers to the tendency to remember the first and last items in a list better than the ones in the middle. While there is a predictable rate at which information is lost from short-term memory, it is not relevant to this question. Similarly, while the cerebellum plays a role in procedural memory, that fact is not the focus of this question. Finally, hindsight bias, the tendency to think you knew an answer all along after hearing it, does not relate directly to this question.

44.(C) Erikson’s initiative versus guilt stage is most closely related to Freud’s phallic stage. Both occur between the ages of roughly 3 to 5 and are the third stage in their respective theories. Freud’s oral stage corresponds to Erikson’s trust versus mistrust stage. Freud’s anal stage corresponds with Erikson’s autonomy versus shame and doubt stage. Freud’s concept of latency is akin to Erikson’s industry versus inferiority stage. While Freud’s final stage is the adult genital stage and it lasts from puberty on, Erikson divided adolescence and adulthood into several stages: identity versus role confusion, intimacy versus isolation, generativity versus stagnation, and ego integrity versus despair.

45.(D) Daniel is most likely to be diagnosed with autism, an impairment of social development. Common symptoms include delayed speech, avoiding eye contact, and a preference for routine. A description of Down syndrome, or trisomy 21, is likely to mention an extra chromosome on the 21st pair and a degree of intellectual impairment. A description of fetal alcohol syndrome would probably include mention of a mother who drank while pregnant and would likely also mention intellectual impairment. The hallmark of mental retardation is intellectual functioning that is significantly below average. Klinefelter’s syndrome results from a chromosomal abnormality of the sex chromosomes; boys with Klinefelter’s syndrome have an XXY genotype. Signs of Klinefelter’s syndrome include small testes, atypical secondary sex development, and infertility.

46.(E) Approximately 95 percent of the population scores between 70 and 130 on the WISC. The WISC has a mean of 100 and a standard deviation of 15, which means that the scores between 70 and 130 represent all the scores within two standard deviations of the mean. By definition, in a normal distribution, about 95 percent of scores fall within two standard deviations of the mean.

47.(B) The school librarian’s system of giving students a prize for every five books they read uses a fixed ratio reinforcement schedule. The schedule is fixed (as opposed to variable) because students are rewarded for every five books they read—a fixed or constant number. The schedule is a ratio (as opposed to interval) schedule because what controls the reinforcement is the number of responses the person makes and not the passage of time.

48.(A) The fact that students read more books in the summer but fewer books in the fall is probably due to the overjustification effect, the finding that extrinsic rewards can undermine intrinsic motivation. Students who previously read because they liked to read may have come to think they read for the prizes and not because they liked reading. As a result, when the prizes are withdrawn, they no longer had a reason to read and therefore may have read less.

49.(E) It is impossible to prove a theory. Experiments and other research methods, even when they yield statistically significant results and those results are replicated, can garner support for a theory but not proof. It isn’t really even possible to prove a hypothesis because the term proof implies that you are absolutely sure about something and given the role of probability in hypothesis testing, we can never really be 100 percent certain about anything.

50.(B) Lupe’s situation with Craig represents an approach-avoidance conflict. She is attracted to certain aspects of Craig but not to others. In an approach-approach conflict, one has to choose between two desirable options—for instance, if Lupe was attracted both to Craig and his brother Greg. In an avoidance-avoidance conflict, one has to choose between two undesirable options; imagine Lupe’s parents and Craig and Greg’s parents had agreed their children would wed. If Lupe disliked both Craig and Greg but had to marry one of them, she would face an avoidance-avoidance conflict. In a multiple approach-avoidance conflict, one has to choose between several options, each of which has attractive and unattractive features. If Lupe thought Craig was attractive but boring and Greg was interesting but not attractive to her, she would face a multiple approach-avoidance conflict.

51.(D) Ani’s belief that her attitudes and behavior play a central role in what happens to her is associated with an internal locus of control. While people with an external locus of control feel fate and luck play a large role in their lives, people with an internal locus of control believe they control their destinies. A strong superego is associated with a being concerned with morals and ethics. People with low self-esteem have negative thoughts about themselves. People with low self-efficacy doubt their ability to get things done. Extraverted people are friendly and outgoing.

52.(C) The fact that people’s ears are located on opposite sides of their heads is most adaptive because it helps us identify the origin of a sound. We locate sound by comparing the time it takes a sound wave to reach each ear and the intensity of the sound wave when it hits each ear. Sounds that come from our right reach our right ears more quickly and with greater intensity than sounds that come from the left and vice versa. The location of our ears does not increase the range of frequencies we can hear or help us gauge sound intensity, respond to noises behind us, or figure out what sounds to ignore.

53.(D) Young Tina had never seen the space shuttle until her parents pointed out a picture of it ready to launch. When she next saw a picture of it flying, she had difficulty recognizing it. Tina’s problem can best be explained by shape constancy. Shape constancy is a perceptual constancy that we learn from experience. An unfamiliar object looks different from different angles, and we have to learn that it is the same. Until we do, we make errors like Tina’s. The autokinetic effect is when a spot of light in a dark room appears to move on its own. Dishabituation refers to an increase in response that occurs to a novel stimulus. Summation refers to the way the layers of the retina are set up: multiple rods and cones synapse with one bipolar cell and multiple bipolar cells synapse with a single ganglion cell. Egocentrism is one of the cognitive limitations of the preoperational child, according to Piaget.

54.(D) Sal’s conclusion that Petunia is a funny person after their brief meeting is best explained by the fundamental attribution error, the tendency for people to underestimate the role of the situation in explaining the behavior of others. The mere-exposure effect would hold that the more time Sal spent with Petunia, the more he would like her; it says that exposure to a person or thing increases liking. Self-serving bias is the tendency to take more credit for positive outcomes than negative ones. Equipotentiality is a learning principle, the belief that we have an equal opportunity to teach all organisms all things. Cognitive dissonance is the mental tension that arises from holding two contradictory thoughts or when people’s thoughts don’t mesh with their behavior.

55.(B) Since Edy is giving the cookies out herself and they have her name on them, the act of distributing will increase the student body’s familiarity with her. According to the mere-exposure effect, increased exposure increases liking. Foot-in-the-door is a compliance technique that involves getting someone to agree to a small request in order to increase the likelihood they will agree to a larger, subsequent request. Edy’s technique utilizes the peripheral route more than the central route; the central route to persuasion would involve Edy explaining why she is the best candidate. Pluralistic ignorance is one explanation for the bystander effect. Deindividuation is a loss of self-restraint under conditions of heightened arousal and relative anonymity.

56.(E) Alyssa is testing the impact of framing; milk that is 1 percent fat is the same as milk that is 99 percent fat free; the only difference is how the milk is framed. Representativeness heuristic is when people judge the likelihood of an event by comparing it to something they believe to be similar and assuming the probabilities of the events will be the same. Confirmation bias is the tendency to pay more attention to information that supports our preexisting beliefs than information that contradicts it. Schema are cognitive structures that influence how we process information. The mere-exposure effect shows that exposure to something increases our liking for it.

57.(C) Eli, if developing typically and 6 months old, is probably just beginning to sit up. Infants develop in a predictable sequence and a fairly predictable rate: at about one month, most infants can lift their head; by 3 months, most can hold their heads steady; by 6 months, most can sit up; by 9 months, most are beginning to try to stand up while holding on to something; and by a year, most infants begin to stand on their own and take a step or two.

58.(A) Harlow’s monkey experiment illustrated the importance of physical contact to development. Harlow compared infant monkeys’ attachment to a surrogate mother made out of cloth or one made of wire that was attached to a bottle so “she” could feed the baby. He found the infant monkeys preferred the soft “mothers.” His study did not involve language or tools. Even though the infant monkeys were separated from their natural parents, the goal was not to study the impact of that separation, and the study also did not look at the heritability of shyness.

59.(B) Disorganized language including the use of clang associations and neologisms is a common symptom of schizophrenia. These communication problems are thought to reflect disordered thought.

60.(E) In correlating the two sets of results, Jupiter is measuring the test’s reliability. This type of reliability is known as test-retest reliability. Since perfectionism should not change over the course of the month, a low correlation would indicate that the test was not reliable. Outliers are extreme scores, and Jupiter does not appear to be looking for them. To standardize the test, Jupiter would have to give it to a standardization sample and then look to assemble a group of questions the yielded a normal distribution of scores. Nothing in the question suggests Jupiter believes the mean level of perfectionism should have changed, and the correlation will not necessarily show whether it has. Finally, even though Jupiter’s work will show whether or not the test is reliable, it will not show whether or not it is valid. Validity is a measure of accuracy—in this case, whether the test actually measures perfectionism.

61.(B) Bandura showed that people learn by observing and then imitating others, a phenomenon known as modeling. Instrumental learning, named by Thorndike, is when people or animals learn that their behaviors are instrumental in bringing about specific consequences. The copycat effect is a term made up to sound like it fits the behavior described but not, in fact, used to do so. Thanatos is the name Freud gave to the supposed death instinct that drives people to commit aggressive and destructive acts. Sublimation is a Freudian defense mechanism in which people take the energy from impulses they view as unacceptable and redirect them toward more socially acceptable pursuits.

62.(C) The technique Coach Perry is using is called shaping. Shaping is defined as rewarding successive approximations of a desired behavior. Coach Perry would love 5-year-old Lana to be able to stop his hardest kick, but he knows that in order to reach that goal he must begin by letting her stop slow-moving balls. The law of effect is Thorndike’s pronouncement that pleasant consequences will increase the likelihood of a behavior and unpleasant consequences will decrease the likelihood of a behavior. The partial reinforcement effect is the finding that partial reinforcement schedules are more resistant to extinction than continuous reinforcement. Second-order conditioning is when, in classical conditioning, you use something that was initially a CS as a US to condition a new CS. A token economy is a method of promoting desired behaviors by rewarding such behaviors with tokens, redeemable at a future time for any of a range of items.

63.(C) A p value of .05 or less indicates that a difference is statistically significant. p stands for probability, and a p value gives the probability that the result occurred by chance. To compute the p value in this case, Matt will have to use both the standard deviation and the mean, but neither of those alone will tell him whether or not the difference is significant. Given the type of data with which Matt is working, he would have to run a t-test rather than a chi square test or a Pearson correlation to find a p value.

64.(B) Matt’s research design is best classified as an ex-post facto study. In an ex-post facto study, what the researcher conceives of as the independent variable has already been predetermined. Student gender is predetermined; Matt cannot randomly assign students to be either girls or boys; hence, the study is an ex-post facto study. Random assignment of participants is required in an experiment. A case study looks closely at only one or a small group of people. Naturalistic observation uses unobtrusive observation rather than a survey. A content analysis studies the coverage of various topics in a communications medium (e.g., television, textbooks).

65.(D) The population in Matt’s study is the Northeastern High School student body. The population is defined as anyone who could have been in the study. Since Matt took a random sample of the whole student body, they all had a chance to be in the study. The group of 100 students who actually took the survey is known as the sample. American students, high school students in Maine, and high school students throughout the world are all groups that Matt might be interested in, and they might be closer to what you think of when you think of the word population, but they are not the population in this study because no one who was not attending Northeastern High School had the chance to be in Matt’s study.

66.(A) The part of the brain thought to play the most important role in the Cannon-Bard theory of emotion was the thalamus. These psychologists thought the thalamus received information from the environment and simultaneously sent signals to the cortex and autonomic nervous system.

67.(A) Selye’s general adaptation syndrome explains how the body deals with stressors. It has three stages: alarm, resistance, and exhaustion. After working so hard to deal with the stress of finals, Conrad reached what Selye termed exhaustion and succumbed to illness. The Yerkes-Dodson law explains the relationship between arousal and performance. The law of effect explains the relationship between the consequences of an action and repetition of that action. Cognitive dissonance theory proposes that we are motivated to maintain consistent attitudes. The James-Lange theory of emotion states that specific physiological changes in the body are indicative of specific emotions.

68.(C) Learned helplessness is most likely to be linked to an external locus of control. Learned helplessness is when exposure to circumstances one cannot control leads one to believe that one cannot control later events that are, in fact, within one’s control. Similarly, someone with an external locus of control believes that factors such as luck and fate determine what happens to them as opposed to internal factors like effort or skill. The Barnum effect is that people believe vague, stock personality descriptions describe them. Self-serving bias is the tendency to take more credit for positive outcomes as opposed to negative ones. You have unconditional positive regard when someone accepts you regardless of your faults; humanistic theorist Carl Rogers thought it essential for a person to be psychologically healthy. An Oedipal complex is the Freudian idea that boys are in love with their mothers.

69.(C) Jung’s collective unconscious contains archetypes, which are universal thought forms he believed were passed down through the species. Fictional finalisms and feelings of inferiority are terms more closely associated with Alfred Adler, another neo-Freudian. Jung believed complexes and bad memories would be stored in the personal unconscious.

70.(B) The shortest electromagnetic waves humans can see are violet. The acronym Roy G. Biv makes it easy to remember the order of the colors in the visible spectrum. The red waves are the longest followed by orange, yellow, green, blue, indigo, and finally violet.

71.(B) People who are able to be hypnotized easily are likely to score high on the openness dimension of the big five personality traits. Openness means a willingness to try new experiences and a certain lack of cynicism that is probably linked to hypnotic suggestibility. How agreeable (easy to get along with), conscientious (hardworking), emotionally stable (even-keeled), and extraverted (outgoing) a person is are less likely to be related to hypnotic suggestibility.

72.(C) The right hemisphere controls the left side of the body and is important in recognizing faces, doing spatial tasks, and all sorts of creative pursuits. The left hemisphere plays a larger role in speech and language, logical problems, and controlling the right side of the body. Plasticity is the brain’s ability to change as the result of experience, and one way it is illustrated is when the functions of a damaged part of a brain are taken over by another part of the brain. Since most of Genna’s right hemisphere was damaged and removed, plasticity is shown by her ability to perform functions associated with that hemisphere after the surgery. The only behavior associated with the right hemisphere is moving her left hand.

73.(D) One of the more impressive feats cited by those who believe that animals are capable of true language is that some apes have been able to use words they were taught to express novel concepts. For instance, Washoe, who didn’t know the word for refrigerator, was able to describe it as a “cold box.” Chomsky speaks of a language acquisition device, but the term does not refer to a physical structure in the brain. Even though primates are able to learn words that result in food rewards, critics of the idea that animals have language say these words are merely the result of operant conditioning and do not evidence true language. There is no doubt that animals including honeybees and dolphins can communicate with one another and with people to some extent; however, there is much debate over what should be considered “language.” Young apes are much slower to pick up new vocabulary than are young humans.

74.(A) The finding that people who read a job description written only with male pronouns (e.g., he) are more likely to think of an employee as male than people who read a description that uses gender-neutral language (e.g., he or she) is most closely linked to the linguistic relativity hypothesis. The linguistic-relativity or Sapir-Whorf hypothesis holds that language influences thought. Therefore, people who read a job description written using only male pronouns are influenced to think of the person who holds the job in a particular way. Gender schema theory holds that children learn about the gender roles of their culture and that the expectations they develop then guide their behavior. The social role hypothesis is one explanation for hypnosis; specifically, this hypothesis says that hypnotized people play the role that is expected of them and do not actually experience an altered state of consciousness. Modeling, or observational learning, is Bandura’s theory that people learn by watching and imitating others. Ethnocentrism is the tendency to look at things from the perspective of one’s own culture.

75.(D) In the nature versus nurture debate, nature is most closely associated with temperament. Nature refers to things that are innate. Temperament is one’s inborn style of interacting with the world. Most of the other terms—environment, culture, and learning—are associated with nurture. One’s family plays a role in both nature (for example, via genes) and nurture (e.g., via the home environment).

76.(D) If the College Board neglected to put any questions that had to do with neuroscience on the AP Psychology exam one year, the test would lack content validity. A valid test measures what it is supposed to measure. A test with high content validity covers all the areas it is supposed to in the appropriate level of depth. Predictive and concurrent validity are two types of criterion validity. A test with predictive validity provides a measure of how someone will perform on a task in the future, whereas concurrent validity provides a measure of how someone will perform at the present time. Face validity is a shallow type of content validity; if a test is supposed to assess mathematical skill and contains math problems, it has face validity.

77.(D) Kohler’s studies with apes involved setting up various problems to be solved such as how to retrieve bananas positioned out of reach. Kohler’s observations led him to argue that the animals solved the problems by thinking about them—specifically, by having sudden insights about how to solve them.

78.(C) The scores of the three students who score much differently from the rest of the class are known as outliers or extreme scores. The mode is the most frequently occurring score in a distribution. Even though outliers skew a distribution, they are not called skewers. Variance is a measure of the variability in a distribution as is standard deviation.

79.(D) Positive psychology focuses on the study of subjective well-being, optimism, and happiness. It is a relatively new area of psychology that grew out of the belief that too much of psychology focused on the negative aspects of the human condition (e.g., depression, prejudice, aggression). Psychometrics is the study of how to make tests. Human factors psychology melds psychology and engineering in an effort to make machines that are user-friendly. Social psychology is the study of how people interact with one another, and health psychology is an area that looks at how psychological factors impact physical health.

80.(D) After taking AP Psychology and doing well in the class and exam, Donald goes to college. If Donald is interested in psychology and has high achievement motivation, as a first year student, he is most likely to sign up for an upper-level undergraduate course in which he will have to work hard to succeed and will learn new things. His interest in psychology will lead Donald to want to take more psychology classes and learn new things, while his high achievement motivation will lead him to select a class that will be challenging but not overly difficult.

81.(A) Humanistic models of personality reject the idea of determinism (that one’s action and choices have been determined by factors outside of one’s control) and embrace the idea of free will (that people are able to make their own choices). Biological models of personality tend to see human thought and behavior as determined by physiological factors. Evolutionary models view genes as the determining factor over most human behavior. Behaviorists believe one’s history of reinforcement determines future choices, and psychoanalysts see the thoughts in one’s unconscious as the factor that controls human behavior.

82.(B) Neuroleptics are commonly prescribed to treat schizophrenia, the most common psychotic disorder. L-Dopa is used to treat Parkinson’s disease and would likely have an adverse effect on Vance since it would lead to an increase in dopamine and an excess of dopamine is a hypothesized cause of schizophrenia. Benzodiazepines are commonly used to treat anxiety. SSRIs (serotonin selective reuptake inhibitors) are used to treat major depressive disorder, and lithium is used to treat the manic phase of bipolar disorder.

83.(B) Sperling’s partial report technique was designed to test the capacity of sensory memory. Sperling hypothesized that the reason people were only able to report about four items they were exposed to for a split second was not that more items didn’t make it into the sensory register but that the memory decayed before the people could report them. He used the partial report technique to expose participants to a matrix of 12 letters (3 rows of 4) and showed that if cued on which line to report immediately after the matrix was removed from view, participants could usually recall the entire row correctly. This technique is not used to test the serial position effect, duration of working memory, difference between STM and LTM, or misinformation effect.

84.(C) Parkinson’s disease has been linked to a deficit of dopamine. Major depressive disorder is associated with low levels of both serotonin and norepinephrine. The cause of autism is currently unknown. Similarly, the role of various neurotranmitters in Alzheimer’s disease remains debated, but one view holds that shortages of acetylcholine are involved. Schizophrenia is associated with elevated levels of dopamine.

85.(A) Runners in a park were found to pick up their pace when another runner came into view; this finding illustrates the phenomenon of social facilitation since it shows that the presence of others improved performance. Conformity involves a change in attitude or behavior to fit in with a group; if a runner ran faster to keep up with her soccer teammates, conformity could be at work. Deindividuation is a loss of self-restraint under conditions of high arousal and anonymity. Norms are standards of expected behavior, and roles are sets of expected behaviors for different members of an organization.

86.(E) The Rosenhan study of mental institutions showed that confirmation bias, the tendency to pay more attention to information that supports one’s views than challenges them, may influence clinicians’ views and treatments of mental patients. In this study, a number of people checked themselves into a variety of mental institutions, including small private facilities and large public ones, reporting that they heard voices saying “empty,” “hollow,” and “thud.” The otherwise normal people were admitted and then resumed their normal behavior. The study revealed that it took a long time for any of the hospitals’ employees to realize that these patients were not in need of treatment, something that might be partly explained by the fact that the staff members tended to avoid contact with the patients. When the pseudopatients were released from the hospitals, they were diagnosed as schizophrenics in remission, which illustrates the stickiness of labels. The Rosenhan study showed that it can be difficult to discern who is mentally ill and who is well.

87.(B) An American teenager’s prototype of a chair is most likely to include four legs and a seat. A prototype is the most typical example of category. Although there are many types of chairs such as the armchair, chairlift, and wheelchair listed in choice D, most chairs we encounter have the features described. We may commonly think of chairs with desks and tables or as a place to keep our pens, pencils, books, and computers. Through classical conditioning, someone may have come to associate certain types of chairs with anxiety.

88.(C) According to Kohlberg, in order to reason at a high moral level, people must be able to take another person’s perspective. Kohlberg’s theory includes three levels of moral development: preconventional, conventional, and postconventional. One of the hallmarks of postconventional thought is the ability to see things from multiple perspectives. Although we think of generosity as a good thing, it is conceivable that someone in the postconventional stage would decide that generosity, at least in some situations, should be avoided. Kohlberg’s theory is not linked to any religious beliefs. Erikson’s theory includes the integrity versus despair stage, but this idea is not tied to Kohlberg’s theory. One could come to postconventional morality in a number of ways; one does not have to have learned one’s moral beliefs from good role models.

89.(B) The incidence of schizophrenia in the population is closest to 1 in 100.

90.(A) Learned helplessness, a term coined by Martin Seligman, is when exposure to uncontrollable circumstances leads one to believe that he or she cannot exert control over other, controllable circumstances. Research has shown that learned helplessness is correlated with major depressive disorder.

91.(C) About an hour into one’s first sleep cycle, people are typically in stage 3 or 4. In these stages, they experience deep, slow-wave sleep, which is marked by the presence of delta waves. Alpha waves indicate drowsiness, beta waves are present when one is awake and alert, and sleep spindles and K complex waves occur mostly in stage 2.

92.(B) Information from the eyes’ retinas crosses at the optic chiasm. Each eye sends information to both hemispheres of the brain. Information from the left side of each retina goes to the left hemisphere of the brain, and information from the right side of each retina goes to the right side of the brain. The information is traveling via the optic nerve. The fovea is the part of the retina that has the highest visual acuity. The lateral geniculate nucleus is the part of the thalamus where initial visual processing takes place. The basal ganglia are a group of nuclei that help coordinate different parts of the brain.

93.(D) In studying for the AP Psychology exam, good advice would be to study from multiple sources. Studying from your class notes, homework, old tests, and review book involves a fairly deep level of processing and will result in more elaborative encoding. Short-term memory only lasts about 20–30 seconds, so it’s not a place to store information you’re going to need hours later. Even though we hope you find our book helpful, reading any one source over and over again is a relatively shallow form of processing, and you would do better to study from multiple sources. Since what you need to know for the exam is not a long list of items, the serial position effect would not be particularly helpful. Finally, although minimizing interference is a good goal, you will perform better with a good night’s sleep, and research shows that interference during sleep is actually minimal.

94.(D) A lesson from Janis’s research on groupthink is that it is important for people to voice dissent. Groupthink occurs when dissent in a group is suppressed in order to preserve harmony, and bad decisions result. Classic examples include the Bay of Pigs fiasco and the Challenger explosion. Unanimity, pressure, and homogeneity are all factors that can promote groupthink. Groupthink can occur in groups of any size.

95.(C) Information from the optic nerve is initially processed in the thalamus, in the lateral geniculate nucleus (LGN). From the thalamus, information is sent to the primary visual cortex in the occipital lobe. The hypothalamus, hippocampus, and cerebellum are not thought to play a large role in the early processing of visual information.

96.(B) Cognitive psychologists would be most likely to support Schachter’s two-factor theory of emotion. This theory holds that emotion results from the cognitive appraisal of a general physiological arousal. Social facilitation is the finding that people perform well-learned tasks better in the presence of others. The other three theories listed, while theories of emotion, do not give cognition a prominent role. Cannon-Bard’s thalamic theory posits that the thalamus receives information from the environment and simultaneously sends signals to the cortex and autonomic nervous system. The James-Lange theory suggests that every emotion is associated with a unique physiological set of responses that tells us what emotion we feel. The opponent process theory of emotion states that the experience an emotion triggers is followed by the experience of its opposite.

97.(D) Odette has several classic symptoms of major depressive disorder—loss of appetite, disrupted sleep, and a loss of interest in her usual activities. Although Odette is at an age when many people sadly begin to suffer from Alzheimer’s, she is not experiencing the rapid degeneration in memory and cognitive function associated with that disease. Since her problems began about a year ago, they do not seem to be associated with winter, and she is unlikely to be diagnosed with seasonal affective disorder. There is no indication that she is engaging in the binge-purge cycle associated with bulimia nor is there evidence that Odette evinces the major symptom of antisocial personality disorder, a disregard for and carelessness toward others.

98.(C) One cause of the deinstitutionalization of many psychiatric patients in the mid-1900s was the creation of new medications such as antipsychotic drugs. It was hoped that this movement would save the government money and enable people suffering from mental illness to have more freedom. Unfortunately, once released from the institutions, many people did not stay on their medication regimes, and without adequate systems in place to follow up on these people, many became homeless. Even though methods of psychotherapy may have improved, the former patients no longer had access to it. The deinstitutionalization movement was not the result of a belief that we had institutionalized healthy people.

99.(D) The prefrontal cortex of Phineas Gage’s brain was damaged by his accident while laying the railroad track. This part of the brain plays a vital role in judgment and decision making, and the damage helps explain how Gage’s personality changed from dependable and even-tempered to irascible and irresponsible. The corpus callosum is the band of neurons that links the brain’s two hemispheres. The temporal lobes are located on the sides of the head just above the ears and house the auditory cortex. The medulla oblongata sits atop the spinal cord and helps regulate involuntary activities like heart rate. The midbrain is part of the brainstem and contains the reticular formation, which regulates arousal.

100.(D) Approximately 60 percent of participants in Milgram’s obedience experiment thought they delivered the maximum amount of shock possible, a figure far in excess of what psychologists had predicted. Remember, no actual shocks were delivered; the “learner” was a confederate of the experimenter trained to act the part.

Practice Test 2

ANSWERS EXPLAINED

1.(C) Wundt set up the first psychological laboratory and his empirical, replicable experiments are the reason he is called the father of the science of psychology.

2.(C) Skinner focused on operant conditioning within the perspective of behaviorism, which specifically addresses the impact of reinforcers and punishers on behavior. The other theorists listed did not specifically focus on operant conditioning in their research.

3.(B) Choosing participants at random for a study is random sampling. Assignment occurs when participants are assigned to control and experimental groups. Representative sampling and assignment involve purposefully making groups that represent a larger population.

4.(B) A double blind study controls for both experimenter and subject bias since neither the researchers nor the participants know who is assigned to the experimental and the control groups while the study is taking place. The other concepts listed as possible answers do not relate directly to both experimenter and subject bias.

5.(D) Impulses from the eyes travel to the occipital lobe where they are perceived as vision. If the occipital lobe is damaged, vision would most likely be affected.

6.(A) Low levels of serotonin are associated with clinical depression, so a medication prescribed to treat depression will most likely affect serotonin levels, rather than any of the other neurotransmitters or other options listed.

7.(A) MRI and CAT scans provide detailed information about the structure of the brain (and other body parts). These scans would reveal any structural brain damage caused by the car crash. The PET, EEG, and fMRI scans are primarily used to detect brain function, not structure (although the fMRI would also provide structural details). An EKG scan is unrelated to brain function.

8.(B) The sympathetic nervous system is responsible for the “fight or flight” response—mobilizing our body in response to stress (such as surprise or shock). The other terms in the question do not relate to this response.

9.(E) Dendrites grow more quickly in children, so brains of younger people are more “plastic,” or changeable. A young person who suffers brain damage is more able to make new connections in nondamaged parts of the brain and regain lost functions. The other brain-related concepts listed are not related to brain plasticity.

10.(D) PET scans provide information about which parts of the brain are most active, which would reveal that all parts of the brain are used (although different areas of the cerebral cortex are more active than others on different tasks). CAT and MRI scans only provide information on the structure of the brain, not brain activity. An EEG scan does provide information about brain wave activity, but does not tie this activity to specific areas of the brain. And EKG is a scan of heart function, not related to brain activity.

11.(D) Although all the structures listed are involved in vision (including color vision), cones are uniquely involved. Cones are cells in the retina that respond to wavelengths of light that we perceive as color.

12.(C) Vision, hearing, and touch can be categorized as energy senses, since they gather energy (in the form of either light or sound waves). Taste and smell can be categorized as chemical senses since they gather chemicals (through the mouth or nose) and turn them into smell and taste perceptions.

13.(A) Volume is determined by the amplitude (height) of the sound wave. Wave frequency determines the pitch of the sound.

14.(B) The absolute threshold represents the minimum amount of stimulus we can usually detect, such as the minimum amount of salt we can taste in water.

15.(D) Detecting the difference between two very similar pitches depends on the difference threshold, the minimum difference between two stimuli that we can perceive.

16.(C) Depth perception depends on monocular cues (like linear perspective) and binocular cues (like retinal disparity).

17.(B) People deprived of REM sleep have difficulty with memory tasks. REM sleep deprivation does not increase stage 3 and 4 sleep (in fact, people deprived of REM sleep more easily slip into sleep onset and REM sleep, not deeper sleep stages like stage 3 and 4). REM deprivation is not related to sleep apnea, night terrors, or manifest dream content.

18.(B) Role theory explains that hypnosis works because people play the “role” of a hypnotized person. Highly suggestible people are more easily hypnotized and have richer fantasy lives. Dissociation theory explains that hypnosis works by actually dividing our consciousness. One part of our consciousness can be aware and respond to the hypnotist while another level maintains awareness of reality. The other concepts mentioned as answers are not related to hypnosis.

19.(C) Classical conditioning always starts with pairing a conditioned stimulus (like a bell) with an unconditioned stimulus (like food). All the other possible answers involve reinforcements and punishments, which are involved in operant conditioning, not classical conditioning.

20.(B) A course grade is a secondary reinforcer because (most of us) learn to value high course grades. We do not need to learn to value primary reinforcers because they are related to basic needs and are rewarding (e.g., food, water, sleep).

21.(C) Observational learning theorists, like Albert Bandura, were primarily concerned with how we learn through observing the actions of others (rather than learning through direct rewards and punishments).

22.(D) In the three-box model, sensations are stored first in sensory memory. Sensations are only stored in sensory memory for a split second and then are either encoded into short term memory or are forgotten.

23.(B) Long-term potentiation is the process of strengthened connections between brain neurons. After repeated firings, the connection is strengthened, and the receiving neuron becomes more sensitive to the neurotransmitters from the sending neuron.

24.(D) The average capacity of short-term memory is about seven items. Grouping items into groups (chunking) increases the capacity of short-term memory.

25.(B) Recognition is the process of matching a current stimuli to something already in memory (e.g., seeing someone and recognizing whether you’ve ever seen her or him before). Recall is retrieving information from memory with an external cue (e.g., looking at someone and remembering her or his name).

26.(C) Researchers (like Elizabeth Loftus) demonstrated that leading questions can cause us to incorporate false details into our memories of events. For example, a question like “How fast was the car going when it went through the red light?” might cause a person to incorporate a red light into the memory even if a red light wasn’t actually present.

27.(B) Maslow’s hierarchy of needs predicts that people are motivated to achieve the next step in the hierarchy of needs. The order of the hierarchy is physiological needs, safety needs, love needs, esteem needs, and finally self-actualization.

28.(D) Stimulation of the lateral hypothalamus causes an organism to be hungry, and stimulation of the ventromedial hypothalamus causes an organism to stop eating. The other brain structures listed are not centrally involved in hunger motivation.

29.(D) Intrinsic motives (such as enjoyment or satisfaction) are associated with behaviors that continue over a long period of time. Extrinsic motivations (e.g., rewards, such as money) are effective in the short term, but behaviors slow down or stop after some time since most extrinsic rewards are temporary.

30.(D) Cross-sectional studies use groups of subjects of different ages in order to infer the impact of age on a variable. Longitudinal studies follow one group of people over a long period of time in order to infer the impact of age on a variable.

31.(E) Infants are born with the rooting reflex, which causes baby to turn her or his head toward something that touches the cheek. This reflex can help a baby find the mother’s nipple in order to eat.

32.(D) Ainsworth categorized three different types of attachment (secure, avoidant, and anxious/ambivalent) by observing infant reactions after parents left and returned while the infant was in a “strange situation” (a room the infant had not been in before).

33.(B) Erikson examined the major social experiences at all stages of life and theorized how these major experiences (which he described in eight stages) impact personality.

34.(C) Children in the concrete operational stage of development understand concepts of conservation (that the physical properties of object, such as number, area, and volume) do not change when objects are rearranged or reshaped. Children in the pre-operational stage do not understand these concepts and will perform differently on related tasks.

35.(C) Freud believed that unconscious conflicts (such as fixations and defense mechanisms) determined our personalities.

36.(A) Many current personality trait theorists believe that our personalities can be described through a combination of the Big Five personality traits: extraversion, agreeableness, conscientiousness, openness to experience, and emotional stability.

37.(B) Trait theorists believe that personalities can be described through a combination of a small number of factors, or traits. Trait theorists often use pencil and paper tests to assess personality, and the results of these tests indicate that the person falls in a specific personality category.

38.(C) Maslow and the humanists believed that all people are working toward self-actualization and will become more mentally healthy if they are given the right environment. Maslow and the humanistic therapists provided unconditional positive regard to their clients in order to help clients discover for themselves how to best move toward self-actualization.

39.(D) Psychoanalysts might use a projective test in order to try to uncover unconscious conflicts and motive. The theory of the projective test is that a person will “project” his or her unconscious conflicts and desires on to vague and ambiguous stimuli (such as ink blots).

40.(B) Achievement tests measure what a person has learned, either knowledge or skill. IQ tests and most entrance exams are aptitude tests, measuring ability or potential (such as the potential to complete law school successfully). Personality tests and projective tests are neither aptitude or achievement tests: they both measure aspects of a person’s personality (either conscious or unconscious).

41.(A) Fluid intelligence is the ability to solve novel, abstract problems. A new logic-based computer game probably depends on this kind of intelligence, rather than crystallized intelligence, which is the ability to use previously acquired knowledge to solve problems.

42.(B) IQ tests are standardized. Test items and test administration are kept standard for different groups of test takers and IQ test results are compared to a norm group (a standardization sample).

43.(D) The symptoms described best match PTSD, post-traumatic stress disorder.

44.(E) Somatoform disorders, like hypocondriasis and conversion disorder, are manifestations of psychological stresses/problems through physical problems.

45.(D) Psychogenic amnesia is categorized as a dissociative disorder. All dissociative disorders involve disruptions in our consciousness, such as the loss of memory that occurs during amnesia.

46.(D) People with depression often have low levels of serotonin, and drugs used to treat depression usually affect the serotonin system in the brain.

47.(C) Schizophrenia is associated with hallucinations and delusions. Both schizophrenia and dissociative disorders involve splits from reality, and both disorders are very disruptive and may interfere with a career and jobs.

48.(A) Psychotherapies all involve talk therapy, rather than behavioral or biomedical (somatic) treatments.

49.(D) Psychoanalysts, like Sigmund Freud, feel that dreams consist of symbols that reveal unconscious conflicts. This kind of analysis is seen as useful because psychotherapists believe that personality and personality difficulties are caused by unconscious conflicts.

50.(E) Somatic therapists view psychological disorders as biomedical issues, caused by either genetic or brain chemistry issues. Somatic therapists are likely to use chemotherapies (using psychoactive medications) rather than any of the behavioral or cognitive therapies listed.

51.(E) Lobotomies were used at one time to treat a variety of disorders, but this kind of serious psychosurgery was stopped when drug therapies because more common and precise.

52.(B) Psychiatrists are required to train for a medical degree and can prescribe medication. The other kinds of therapists listed do not necessarily receive medical training and are less likely to be able to prescribe medication.

53.(C) Including the free address labels is an example of norm of reciprocity, one of the compliance strategies. People are more likely to help if they feel someone has done them a favor, such as including a gift in a letter asking for contributions.

54.(B) Self-fulfilling prophecies occur when our preconceived ideas about someone influences the ways we act toward them, which may increase the likelihood that our preconceived ideas about the person may seem to be confirmed.

55.(B) When people commit the fundamental attribution error, they ignore the situational factors that may influence another person’s behavior and instead attribute the person’s behaviors to their inner disposition.

56.(D) We tend to see members of our own group (the in-group) as more diverse and more favorable than people outside our group (the out-group). This bias could contribute to discrimination and prejudice.

57.(A) Superordinate goals (goals that all groups need to work together on to accomplish) reduce tensions between groups. The other concepts listed do not relate to group tensions.

58.(D) Milgram’s participants believed they were delivering shocks to strangers during his obedience studies. Some researchers think the participants were at serious risk for potential psychological harm because the study indicated they would have harmed a stranger by obeying an authority figure.

59.(A) Psychoanalysis is based on the study of the unconscious mind, a part of our consciousness that we are not consciously aware of. The other possible answers are not unique to psychoanalysis as compared to the other perspectives.

60.(C) The median is the measure of central tendency least affected by extreme scores (or outliers). Outliers can dramatically impact the mean. Standard deviation and range are measures of variability of the distribution. The normal curve is a graph of a normally distributed set of scores.

61.(B) Inferential statistics are used to examine distributions of scores in order to find statistically significant differences. Descriptive statistics (like standard deviation) describe sets of scores, but used alone cannot help make judgments about the significance of differences between sets of scores. Field experiments and counterbalancing are terms related to research methodology and aren’t relevant to the question.

62.(C) Chemicals (neurotransmitters) move between neurons. These neurotransmitters either increase the likelihood the next neuron will fire (excitatory neurotransmitters) or decrease the chance the next neuron will fire (inhibitory neurotransmitters).

63.(C) Broca’s area of the brain (located in the left frontal lobe in most people) helps control the muscles in the jaw, throat, and tongue needed to produce speech. Wernicke’s area is also involved in spoken language but is involved with meaning and interpretation of language. The hippocampus and amygdala areas of the brain are not involved in spoken language. Gage’s area is not an area of the brain (Phineas Gage was a famous case study in brain function, but is not related to a specific area of the brain).

64.(E) The medulla is located at the top of the spinal cord where it enters the brain. This is the area of the brain that controls blood pressure, heart rate, and breathing, and it is in one of the most protected parts of the brain.

65.(B) The doctors who treated Phineas Gage documented what areas of the brain were damaged and how Gage’s physical and emotional characteristics changed after the accident. This was one of the first cases to tie a specific brain area to a specific function.

66.(E) Only monocular depth cues (like linear perspective) can be used in a pencil drawing, since binocular depth cues (like retinal disparity and convergence) depend on the different sensations we receive from two eyes.

67.(D) Generalization occurs in classical conditioning when an organism responds to any stimuli similar to the conditioned stimuli, such as salivating to any sound similar to a bell.

68.(B) Garcia and Koelling’s research in learned taste aversions established that when any organisms become nauseous, they are very likely to associate the nausea with what they just ate or drank, and will avoid that food or drink in the future.

69.(E) Organisms trained using variable ratio schedules of reinforcements will continue to perform the desired behavior for a long time after reinforcement is stopped (i.e., extinction of the behavior is delayed).

70.(B) Semantic memories are general knowledge about the world. Procedural memories are memories of skills and how to perform them, and episodic memories are memories of specific events. Eidetic and mnemonic are not specific kinds of long-term memories.

71.(B) Explicit memories are memories of facts and events. Implicit memories (or nondeclarative memories) are memories of procedures or skills that we may not even realize we have, such as the skill of tying a tie.

72.(C) Research indicates that biological factors, such as hormones released to the developing fetus in the womb, may influence later sexual orientation. Research indicates that the other factors listed as possible answers do not influence sexual orientation.

73.(E) Theory Y managers believe that workers want to do good work and set policies in order to support worker efforts to improve and do quality work. Theory X managers believe that workers will only produce if given rewards and punishments. The other options listed as answers are not associated with management style theory.

74.(C) Motor development occurs in predictable stages, and develop as brain neurons mature and are further myelinated (thus improving communication between neurons).

75.(C) Authoritative parents set and enforce rules for children, but they talk about the reasons behind and the importance of the rules and help children understand the rules and participate in discussions about good and bad behavior.

76.(B) Kohlberg’s theory dealt with the development of moral thinking. He would have been interested in how children of different ages responded to this question about the ethics of lying.

77.(E) Freud’s theory was built on case studies from his psychoanalytic practice and was not tested empirically. In fact, his claims about the unconscious could not be tested experimentally because the unconscious by definition is not accessible to conscious investigation.

78.(A) A person who buckles down and studies more after failing an exam must believe that her studying will benefit her and that she will do better on the next exam because of her efforts. This is the definition of an internal locus of control: the belief that our actions have impact and that we are in control of what happens to us.

79.(D) 100 is the average score on IQ tests, with a standard deviation of 15. A score of 145 is three standard deviations above the average, and is a very high IQ score.

80.(B) In order to be diagnosed with a psychological disorder, a person’s behavior is maladaptive (harmful to themselves or others), disturbing (disturbs others), unusual (atypical, not common), and irrational (not based in reality).

81.(B) In order to be diagnosed with major depression a person has to have a depressed mood that lasts for longer than two weeks without an obvious cause.

82.(B) Groupthink occurs when a group makes a bad decision because members of the group did not want to contradict each other (often due to mutual admiration of group members). Any techniques used to encourage contrary opinions within the group will work against this groupthink tendency.

83.(D) Asch found that conformity increased if the group was made up of three or more people.

84.(D) This psychologist is advising the patient to examine her or his own thinking (cognitive perspective) and prescribing a psychoactive medication (biopsychology perspective).

85.(B) Experiments use experimental and control groups to isolate what happens when a specific variable (independent variable) is changed and to measure the impact on the variable that changes as a result (dependent variable). The other options listed are either not true about the experimental method or are not unique to the experimental method.

86.(D) This study would most likely be done by comparing the average amount spent on organic food between people who own hybird cars and people who don’t. This study could not be performed as an experiment (there is no practical way to randomly assign people to “own hybrid cars” and “don’t own hybrid cars” groups). The other research methods listed would not allow researchers to conclude about the likelihood of spending money on organic food.

87.(B) Purposely exposing middle school students to online bullying, or monitoring private social network interactions, would be unethical. In addition, self-reports about this sensitive topic may not be reliable, and conclusions from a single case would not necessarily generalize to the general population. Therefore, Professor Ek should look at correlations between online bullying and other variables.

88.(E) According to gate-control theory, higher-priority touch sensations (e.g., vigorous itching) will be perceived instead of lower-priority touch sensations (e.g., low-level pain related to the skin rash). So this person does not perceive the low-priority rash pain sensations while she is scratching vigorously (a higher-priority sensation), but the pain returns after she stops scratching.

89.(A) Most drugs produce increased tolerance with repeated uses—the need for increasing amounts of a drug in order to produce the same physiological effects. This tolerance gradually changes the levels of specific neurotransmitters in the brain, so when a person stops using the drug, withdrawal symptoms occur as the body and brain readjust and compensate for altered levels of these neurotransmitters. The other concepts listed in the answers are not related to the tolerance-withdrawal cause-effect cycle.

90.(B) Classical conditioning involves pairing conditioned stimuli with unconditioned stimuli, producing a conditioned response. Operant conditioning involves providing a stimuli (a reinforcer or a punishment) after a specific response is performed. The other possible answers provided about classical and operant conditioning are not accurate.

91.(E) Punishments are defined as stimuli that decrease the likelihood that an organism will perform the behavior that preceded the punishments. Negative reinforcements reinforce a behavior, increasing the likelihood the behavior will be repeated. They reinforce the behavior by taking away an aversive stimulus (e.g., an aspirin takes away a headache, which makes the person more likely to take an aspirin in the future).

92.(D) Chomsky and Skinner disagreed about how children acquire language. Skinner’s behaviorist theory held that children learn language like they learn everything else: through rewards and punishments. Chomsky pointed out that language acquisition occurs too quickly to be explained by reward and punishment. Chomsky hypothesized that humans must be born with a language acquisition device that enables us to learn language quickly during a certain window of opportunity during childhood.

93.(C) Drive Reduction theory states that we are motivated by primary drives (like hunger) and secondary drives, and that we act to satisfy these drives until we regain a state of homeostasis (balance—in this case, we are no longer hungry). The other options each relate to a different motivation theory.

94.(E) Options A and B describe James-Lange’s and Cannon-Bard’s theories of emotion, respectively. Schacter’s two-factor theory holds that a combination of biological changes and mental interpretations (cognitive labels) combined are what we call “emotion.”

95.(A) Piaget described schemas (the ways we think about the world) as going through a process of assimilation and accommodation. When we encounter something new in the world, we first try to use our existing schemas to understand it (assimilation). If that doesn’t work, we may have to change or expand our schemas in order to deal with the new object, event, or idea (accommodation). We learn through accommodation, as our schemas change and become more sophisticated.

96.(D) The social cognitive theorists described personality through reciprocal determinism, meaning that our personality traits interact with our environment, and these both interact with our behaviors. Each of these factors influences the others, and our personality is a result of these interactions.

97.(D) Test-retest reliability is a measure of a test’s ability to deliver similar results each time it is administered to the same person.

98.(E) A test can be reliable (can provide the same results each time it is administered) but may not be valid (those consistent results may not accurately measure what the test claims to measure).

99.(C) The DSM lists the official names of diagnoses and exact symptoms associated with these diagnoses so that psychologists and psychiatrists can reliably diagnose patients. The DSM does not address causes of psychological disorders.

100.(E) Patients using systematic desensitization try to replace anxiety with relaxation. Patients place themselves in somewhat stressful situations and use relaxation techniques (calming thoughts, deep breathing, etc.) to reduce their stress reactions until they are no longer stressed in this situation. Patients then move up to the next level of the “fear hierarchy” and place themselves in a slightly more stressful situation and use the relaxation techniques. The other therapies listed are either strictly behavioral (counterconditioning) or cognitive (rational emotive behavior therapy).